Physics%–%1st%Quarter% - Lorain City School District · PDF file11 L12text’ ......

106
Lorain City School District Scope, Sequence and Pacing Guides Physics Grade 12 1 Common Formative Assessments will be implemented daily. Physics – 1 st Quarter Strand Topic Content Statement Days Clear Learning Target Vocabulary Core Resource Additional Resources Assessment Scientific Inquiry “Motion” Chapter 1: A Physics Toolkit 1. Mathematics and Physics 2. Measurement 3. Graphing Data Chapter 2: Representing Motion. 1. Picturing Motion 2. Where & When? 3. PositionTime Graphs? 4. How fast? Chapter 3: Accelerated Motion 1. Acceleration 2. Motion with Constant Acceleration 3. Free Fall RST.1112.2. Determine the central ideas or conclusions of a text; summarizes a complex concepts, possesses, or information presented in a text by paraphrasing them in simpler but still accurate terms. RST.1112.3. Follow precisely a complex multistep procedure when carrying out experiments, taking measurements, or attempting technical tasks; analyze the specific results based on explanations in the text. RST.1112.9. Synthesize information from a range of sources (e.g., 55 I can demonstrate scientific methods understanding through various scientific phenomenon. I can evaluate answers using dimensional analysis; especially, when dealing with graphs. I can perform arithmetic operations using scientific notation. I can apply data collected using techniques such as CBL. Chapter 1: Physics Dimensional analysis Significant digits Scientific method Hypothesis Scientific law Scientific theory Measurement Precision Accuracy Independent variable Dependent variable Line of best fit Linear relationship Quadratic relationship Inverse relationship Labs #1, 2 and 3 on p.p. (20 21), (48 49), and visit Physics Online for further accelerate Motion. Error! Hyperlink reference not valid. Is a series of videos ondemand produced by Annenberg that show classroom strategies for implementi ng inquiry into the high school This simple animation above depicts some additional information about the car's motion. The velocity and acceleration of the car are depicted by vector arrows. The direction of these arrows is representative of the direction of the velocity and acceleration vectors. Note that the velocity vector is always directed in the same direction which the car is moving. A car moving eastward would be described as having an eastward velocity. And a car moving westward would be described as Standardized Test Practices (1>7) p.p. 29, 55 and 85 Exam 1, 2, and 3

Transcript of Physics%–%1st%Quarter% - Lorain City School District · PDF file11 L12text’ ......

Page 1: Physics%–%1st%Quarter% - Lorain City School District · PDF file11 L12text’ ... Solving’ Many’ problems ... predict’the’effects’of’forces’onmore’complex’systems’of’objects’and’to’analyze’objects

Lorain  City  School  District  Scope,  Sequence  and  Pacing  Guides  

Physics    Grade  12    

1    Common  Formative  Assessments  will  be  implemented  daily.  

 

Physics  –  1st  Quarter  

Strand   Topic   Content  Statement   Days   Clear  Learning  Target   Vocabulary   Core  

Resource   Additional  Resources   Assessment  

Scientific  Inquiry  “Motion”  

Chapter  1:  A  Physics  Toolkit  1.  Mathematics  and  Physics  

2.  Measurement  3.  Graphing  Data  

 Chapter  2:  Representing  Motion.  

1.  Picturing  Motion  

2.  Where  &  When?  

3.  Position-­‐Time  Graphs?  

4.  How  fast?    

Chapter  3:  Accelerated  Motion  

1.  Acceleration  2.  Motion  with  Constant  

Acceleration  3.  Free  Fall  

 

RST.11-­‐12.2.  Determine  the  central  ideas  or  

conclusions  of  a  text;  summarizes  a  

complex  concepts,  possesses,  or  information  

presented  in  a  text  by  paraphrasing  them  in  simpler  but  still  accurate  terms.  

 RST.11-­‐12.3.  

Follow  precisely  a  complex  multi-­‐step  procedure  when  carrying  out  

experiments,  taking  measurements,  or  attempting  technical  tasks;  analyze  the  

specific  results  based  on  explanations  in  

the  text.    

RST.11-­‐12.9.  Synthesize  

information  from  a  range  of  sources  (e.g.,  

55     I  can  demonstrate  scientific  methods  understanding  through  various  scientific  phenomenon.  

 I  can  evaluate  answers  using  dimensional  analysis;  especially,  when  dealing  with  graphs.  

 I  can  perform  arithmetic  operations  using  scientific  notation.  

   I  can  apply  data-­‐collected  using  techniques  such  as  CBL.        

Chapter  1:  Physics    Dimensional          analysis  Significant            digits  Scientific            method  Hypothesis  Scientific            law  Scientific            theory  Measurement  Precision  Accuracy  Independent            variable  Dependent            variable  Line  of  best            fit  Linear      relationship  Quadratic          relationship  Inverse      relationship      

Labs  #1,  2  and  3  on  p.p.  (20-­‐21),  (48-­‐49),  and  visit  

Physics  Online  for  further  accelerate  Motion.  

 Error!  

Hyperlink  reference  not  valid.  Is  a  series  of  videos-­‐on-­‐demand  produced  

by  Annenberg  that  show  classroom  strategies  

for  implementing  inquiry  into  the  

high  school  

 

This  simple  animation  above  depicts  some  

additional  information  about  the  car's  motion.  

The  velocity  and  acceleration  of  the  car  are  depicted  by  vector  arrows.  The  direction  of  these  arrows  is  

representative  of  the  direction  of  the  velocity  and  

acceleration  vectors.  Note  that  the  velocity  vector  is  always  

directed  in  the  same  direction  which  the  car  

is  moving.  A  car  moving  eastward  

would  be  described  as  having  an  eastward  velocity.  And  a  car  moving  westward  

would  be  described  as  

 Standardized  Test  Practices  (1-­‐>7)  p.p.  29,  55  and  85    Exam  1,  2,  and  3    

Page 2: Physics%–%1st%Quarter% - Lorain City School District · PDF file11 L12text’ ... Solving’ Many’ problems ... predict’the’effects’of’forces’onmore’complex’systems’of’objects’and’to’analyze’objects

Lorain  City  School  District  Scope,  Sequence  and  Pacing  Guides  

Physics    Grade  12    

2    Common  Formative  Assessments  will  be  implemented  daily.  

 

texts,  experiments,  simulations)  into  

coherent  understanding  of  a  

process,  phenomenon,  or  concept,  resolving  

conflicting  information  when  

possible.    

RST.11-­‐12.10.  By  the  end  of  grade  

12,  read  and  comprehends  

science/technical  texts  in  the  grades  

11-­‐12  text  Complexity  band  independently  and  

proficiently.    

WHST.11-­‐12.2.  Write  

informative/explanatory  texts,  including  the  narration  of  historical  events,  

scientific  procedures/experim

ent      

I  am  able  to  use  graphing  techniques  to  plot  data,  and  graph  the  relationship  between  independent  vs.  dependent  variables  as  well  as  linear  vs.  nonlinear  relationships.    I  can  distinguish  between  accuracy  and  precision  through  different  examples  proposed  by  teacher.    I  can  determine  the  precision  of  measured  quantities.      

 I  can  interpret  graphs,  and  recognize  common  relationships  in  graphs.    

Chapter2:  Motion            diagram  Practical  Model  Coordinate  System  Origin  Position  Distance  Magnitude  Vectors  Scalars  Resultant  Time            interval  Displacement  Position-­‐        time  graph  Instantan-­‐eous            position  Average              velocity  Average  Speed  Instantan-­‐eous            velocity    Chapter  3:  Velocity-­‐          time  graph  Acceleration  Average    

classroom?  While  not  all  of  the  content  is  aligned  to  physical  

science,  the  strategies  can  be  

applied  to  any  

content.    

www.Physics  

Online.com  

having  a  westward  velocity.  

For  more  information  on  physical  

descriptions  of  motion,  visit  The  Physics  

Classroom.  Detailed  information  is  

available  there  on  the  following  topics:  

Vectors  and  Scalars  

Speed  and  Velocity  

Acceleration  

Describing  Motion  with  Vector  Diagrams  

Motion  Along  Inclined  Planes  

Other  animations  can  be  seen  at  the  

Multimedia  Physics  Studios.  

Use  the  “Interactive  Chalkboard”  using  the  animations,  video,  and  

power  point  presentation  to  bring  

Page 3: Physics%–%1st%Quarter% - Lorain City School District · PDF file11 L12text’ ... Solving’ Many’ problems ... predict’the’effects’of’forces’onmore’complex’systems’of’objects’and’to’analyze’objects

Lorain  City  School  District  Scope,  Sequence  and  Pacing  Guides  

Physics    Grade  12    

3    Common  Formative  Assessments  will  be  implemented  daily.  

 

   acceleration  Instantan-­‐  eous      acceleration  Free  fall    Acceleration  due  to  gravity    

gravitational  theories  and  laws  picture’s  closer  to  students.    Spend  at  most  20  

minutes.  Use  School  Adaptation,  video  tape  to  illustrate  

“Kepler’s  Laws,  Navigating  in  Space,  and  The  Apple  and  The  

Moon.”    

Movie  segments  can  also  be  selected  from:  “Annehttp://www.learner.org/resources/series42.html?pop=yes&pi

d=582”  

https://ieeetv.ieee.org/channels/pels?utm_source=google&utm_medium=cpc&utm_content=Electromagnetism&utm_campaign=IEEEtv+-­‐

+Channels+-­‐+Power+Electronics  

 

 

Page 4: Physics%–%1st%Quarter% - Lorain City School District · PDF file11 L12text’ ... Solving’ Many’ problems ... predict’the’effects’of’forces’onmore’complex’systems’of’objects’and’to’analyze’objects

Lorain  City  School  District  Scope,  Sequence  and  Pacing  Guides  

Physics    Grade  12    

4    Common  Formative  Assessments  will  be  implemented  daily.  

 

Content  Elaborations:  Motion    In  physical  science,  the  concepts  of  position,  displacement,  velocity  and  acceleration  were  introduced  and  straight-­‐line  motion  involving  either  uniform  velocity  or  uniform  acceleration  was  investigated  and  represented  in  position  vs.  time  graphs,  velocity  vs.  time  graphs,  motion  diagrams  and  data  tables.    Forces,  Momentum  and  Motion:  In  earlier  grades,  Newton’s  laws  of  motion  were  introduced;  gravitational  forces  and  fields  were  described  conceptually;  the  gravitational  force  (weight)  acting  on  objects  near  Earth’s  surface  was  calculated;  friction  forces  and  drag  were  addressed  conceptually  and  quantified  from  force  diagrams;  and  forces  required  for  circular  motion  were  introduced  conceptually.  In  this  course,  Newton’s  laws  of  motion  are  applied  to  mathematically  describe  and  predict  the  effects  of  forces  on  more  complex  systems  of  objects  and  to  analyze  objects  in  free  fall  that  experience  significant  air  resistance.    

Strand   Topic   Content  Statement   Days  Clear  

Learning  Target  

Vocabulary   Core  Resource  

Additional  Resources   Assessment  

Scientific  Inquiry  and  Physical  Sciences  

And  Forces,  Momentum  and  Motion  

 

Chapter  4:  Forces  in  One  Dimension  1.  Forces  and  

Motion  2.  Using  Newton’s  Laws  

3.  Interaction  Forces  

 Chapter  5:  Forces  in  Two  

Dimensions  1.  Vectors  2.  Friction  3.  Forces  and  Motion  in  Two  

Dimensions    

RST.11-­‐12.5.  Analyze  how  the  text  

structures  information  or  ideas  into  categories  or  

hierarchies,  demonstrating  understanding  of  the  information  or  ideas.  

 RST.11-­‐12.7.    

Integrate  and  evaluate  multiple  sources  of  

information  presented  in  diverse  formats  and  media  (e.g.,  quantitative  data,  video,  multimedia)  in  order  to  

address  a  question  or  solve  a  problem.    

   RST.11-­‐12.7.  

Integrate  and  evaluate  multiple  sources  of  

information  presented  in  

55  I  can  restate  and  Explain  the  movement  of  object  by  applying  Newton’s  Three  Laws  of  motion.    I  can  apply  principles  of  forces  and  motion  to  mathematically  analyze,  describe  and  predict  the  net  forces  on  objects  or  systems.    I  can  relate  

Chapter  4:  Force  Free-­‐body  diagram  Net  force  Newton’s  Second  Law  Newton’s  First  Law  Inertia  Equilibrium  Apparent  weight  Weightlessness  Drag  force  Terminal  velocity  Interaction  Pair  Newton’s  Third  Law  Tension  Normal  force    

1.  “How  it  Works-­‐bathroom  scale”  p.p.  110  2.  Lab  

Experiment  #  4:  

“Forces  in  elevators”  (Text  Book  p.p.  108-­‐109)        

Problem  Solving  

Many  problems  can  be  

solved  from  interpreting  graphs  and  charts  as  detailed  in  the  motion  graphs  

section.  In  addition,  when  

acceleration  is  constant,  average  

velocity  can  be  

calculated  

Review  packets  (chapters  1,  2,  and  3)  Open  Book  Quizzes  (1,  2,  and  3)  Exams  1,  2,  and  3  

Page 5: Physics%–%1st%Quarter% - Lorain City School District · PDF file11 L12text’ ... Solving’ Many’ problems ... predict’the’effects’of’forces’onmore’complex’systems’of’objects’and’to’analyze’objects

Lorain  City  School  District  Scope,  Sequence  and  Pacing  Guides  

Physics    Grade  12    

5    Common  Formative  Assessments  will  be  implemented  daily.  

 

Chapter  6:  Motion  in  Two  

Dimensions    

1.Projectile  Motion  

2.  Circular  Motion  

3.  Relative  Velocity  

diverse  formats  and  media  (e.g.,  quantitative  data,  video,  multimedia)  in  order  to  

address  a  question  or  solve  a  problem.  

 

and  explain  how  societal  issues  and  considerations  affect  the  progress  of  science  and  technology.    

Chapter  5:  Components  Vector            resolution  Kinetic  friction  Static  friction  Coefficient  of          kinetic  friction  Coefficient  of          static  friction  Equilibrant  Chapter  6:  Projectile  Trajectory  Uniform  circular          motion  Centripetal            acceleration  Centripetal  force    

by  taking  the  

average  of  the  initial  and  final  

instantaneous  velocities  (??𝒂𝒗𝒈  =  (𝒗𝒇  +  𝒗𝒊  )/2).  This  relationship  does  not  hold  true  when  the  acceleration  changes.  The  

equation  can  be  used  

in  conjunction  with  other  Kinematics  equations  to  solve  

increasingly  complex  problems,  including  those  

involving  free  fall  with  

negligible  air  

Page 6: Physics%–%1st%Quarter% - Lorain City School District · PDF file11 L12text’ ... Solving’ Many’ problems ... predict’the’effects’of’forces’onmore’complex’systems’of’objects’and’to’analyze’objects

Lorain  City  School  District  Scope,  Sequence  and  Pacing  Guides  

Physics    Grade  12    

6    Common  Formative  Assessments  will  be  implemented  daily.  

 

resistance  in  which  objects  fall  

with  uniform  

acceleration.  Near  the  surface  of  Earth,  in  the  absence  of  other  

forces,  the  acceleration  of  freely  falling  

objects  is  9.81  m/s2.  Assessments  of  motion  problems,  including  projectile  motion,  will  not  include  problems  that  require  

the  quadratic  equation  to  solve.    

Projectile  Motion  When  an  object  has  

Page 7: Physics%–%1st%Quarter% - Lorain City School District · PDF file11 L12text’ ... Solving’ Many’ problems ... predict’the’effects’of’forces’onmore’complex’systems’of’objects’and’to’analyze’objects

Lorain  City  School  District  Scope,  Sequence  and  Pacing  Guides  

Physics    Grade  12    

7    Common  Formative  Assessments  will  be  implemented  daily.  

 

both  horizontal  and  vertical  components  of  motion,  as  in  a  

projectile,  the  

components  act  

independently  of  each  other.  For  a  projectile  in  the  absence  

of  air  resistance,  this  means  

that  horizontally

,  the  projectile  will  

continue  to  travel  at  constant  speed  just  like  it  would  if  there  were  no  vertical  motion.  Likewise,  vertically  the  object  

Page 8: Physics%–%1st%Quarter% - Lorain City School District · PDF file11 L12text’ ... Solving’ Many’ problems ... predict’the’effects’of’forces’onmore’complex’systems’of’objects’and’to’analyze’objects

Lorain  City  School  District  Scope,  Sequence  and  Pacing  Guides  

Physics    Grade  12    

8    Common  Formative  Assessments  will  be  implemented  daily.  

 

will  accelerate  just  as  it  would  

without  any  horizontal  motion.  Problem  solving  will  be  limited  to  solving  for  the  

range,  time,  initial  height,  initial  

velocity  or  final  

velocity  of  horizontally  launched  projectiles  with  

negligible  air  

resistance.  While  it  is  

not  inappropria

te  to  explore  more  

complex  projectile  problems,  it  

Page 9: Physics%–%1st%Quarter% - Lorain City School District · PDF file11 L12text’ ... Solving’ Many’ problems ... predict’the’effects’of’forces’onmore’complex’systems’of’objects’and’to’analyze’objects

Lorain  City  School  District  Scope,  Sequence  and  Pacing  Guides  

Physics    Grade  12    

9    Common  Formative  Assessments  will  be  implemented  daily.  

 

must  not  be  done  at  the  expense  of  other  parts  of  the  

curriculum.  

Error!  Hyperlink  reference  not  valid.    is  a  series  of  videos-­‐on-­‐demand  produced  

by  Annenberg  that  show  classroom  strategies  

for  implementing  inquiry  into  the  

high  school  classroom.  While  not  all  of  the  content  is  aligned  to  physical  

science,  the  strategies  can  be  

applied  to  

Page 10: Physics%–%1st%Quarter% - Lorain City School District · PDF file11 L12text’ ... Solving’ Many’ problems ... predict’the’effects’of’forces’onmore’complex’systems’of’objects’and’to’analyze’objects

Lorain  City  School  District  Scope,  Sequence  and  Pacing  Guides  

Physics    Grade  12    

10    Common  Formative  Assessments  will  be  implemented  daily.  

 

any  content.  “Annehttp://www.learner.org/resources/series42.html?pop=yes&pid=

582”  

https://ieeetv.ieee.org/channels/pels?utm_source=google&utm_medium=cpc&utm_content=Electromagnetism&utm_campaign=IEEEtv+-­‐+Channels+

-­‐+Power+Electronics  

Content  Elaborations:    Forces,  Momentum  and  Motion:    In  earlier  grades,  Newton’s  laws  of  motion  were  introduced;  gravitational  forces  and  fields  were  described  conceptually;  the  gravitational  force  (weight)  acting  on  objects  near  Earth’s  surface  was  calculated;  friction  forces  and  drag  were  addressed  conceptually  and  quantified  from  force  diagrams;  and  forces  required  for  circular  motion  were  introduced  conceptually.  In  this  course,  Newton’s  laws  of  motion  are  applied  to  mathematically  describe  and  predict  the  effects  of  forces  on  more  complex  systems  of  objects  and  to  analyze  objects  in  free  fall  that  experience  significant  air  resistance.  Gravitational  forces  are  studied  as  a  universal  phenomenon  and  gravitational  field  strength  is  quantified.  Elastic  forces  and  a  more  detailed  look  at  friction  are  included.    At  the  atomic  level,  “contact”  forces  are  actually  due  to  the  forces  between  the  charged  particles  of  the  objects  that  appear  to  be  touching.  These  electric  forces  are  responsible  for  friction  forces,  normal  forces  and  other  “contact”  forces.  Air  resistance  and  drag  are  explained  using  the  particle  nature  

Page 11: Physics%–%1st%Quarter% - Lorain City School District · PDF file11 L12text’ ... Solving’ Many’ problems ... predict’the’effects’of’forces’onmore’complex’systems’of’objects’and’to’analyze’objects

Lorain  City  School  District  Scope,  Sequence  and  Pacing  Guides  

Physics    Grade  12    

11    Common  Formative  Assessments  will  be  implemented  daily.  

 

of  matter.  Projectile  motion  is  introduced  and  circular  motion  is  quantified.  The  vector  properties  of  momentum  and  impulse  are  introduced  and  used  to  analyze  elastic  and  inelastic  collisions  between  objects.  Analysis  of  experimental  data  collected  in  laboratory  investigations  must  be  used  to  study  forces  and  momentum.  This  can  include  the  use  of  force  probes  and  computer  software  to  collect  and  analyze  data.      

Strand   Topic   Content  Statement   Days   Clear  Learning  Target   Vocabulary   Core  

Resource  Additional  Resources   Assessment  

Motion   Chapter  7:  Gravitation  

 1.Planetary  Motion  and  Gravitation  

 2.Using  Law  of  Universal  Gravitation  

 Chapter  8:  Rotational  Motion  

1.Describing  Rotational  Motion  

 2.  Rotational  Dynamics  

 3.Equilibrium  

WHST.11-­‐12.1.  Write  arrangements  focused  on  Discipline-­‐specific  content.  

 WHST.11-­‐12.2.  

Write  informative/explanatory  texts,  including  the  narration  of  historical  events,  scientific  

procedures/experiments,  or  technical  processes.  

   

25   I  can  participate  in  and  apply  the  processes  of  scientific  investigation  to  create  models  and  to  design,  conduct,  evaluate  and  communicate  the  results  of  these  investigations.      I  can  describe  displacement.    I  can  calculate  angular  velocity  and  angular  acceleration.  I  can  solve  problems  involving  rotational  motion    Chapter  8:  1.  I  can  distinguish  between  Angular  velocity  of  an  object  and  Angular  

Chapter  7:  Kepler’s  First            Law  Kepler’s  Second            Law  Kepler’s  Third            Law  Gravitational            force  Law  of  Universal            gravitation  Gravitational                field  Inertial  mass  Gravitational            mass    Chapter  8:  Radian  Angular  Displacement  Angular  velocity  Angular            acceleration  Lever  arm  Torque  Momentum  of            inertia  

Experiment,  Lab  #7.  P.p.  186-­‐187  

of  “Physics  ,  Principles  

and  Problems

”  *Answer  the  

questions  proposed  in  the  

“Analysis,  Conclude  and  Apply  and  Going  Further”  on  

p.p.187  

Error!  Hyperlink  reference  not  valid.    is  a  series  of  videos-­‐on-­‐demand  

produced  by  Annenberg  that  show  classroom  strategies  for  implementing  inquiry  into  the  high  school  classroom.  

While  not  all  of  the  content  is  aligned  to  

physical  science,  the  strategies  can  be  applied  to  any  content.  

 Use  the  

“Interactive  Chalkboard”  using  the  animations,  video,  and  power  point  

Review  packets  (chapters  7  +8)  Open  Book  Quizzes  (7+8)  Exams  (7+8)  

Page 12: Physics%–%1st%Quarter% - Lorain City School District · PDF file11 L12text’ ... Solving’ Many’ problems ... predict’the’effects’of’forces’onmore’complex’systems’of’objects’and’to’analyze’objects

Lorain  City  School  District  Scope,  Sequence  and  Pacing  Guides  

Physics    Grade  12    

12    Common  Formative  Assessments  will  be  implemented  daily.  

 

acceleration  of  an  object  as  the  mathematical  equation  on  p.p.  (198-­‐199).      2.  I  can  draw  Data  Table  8-­‐1  on  p.p.  199  which  supports  my  understanding  to  connecting  “linear  vs.  angular;  and  the  relationships  between  displacement,  velocity,  and  acceleration.”      

 

Newton’s            Second  Law          for  rotational            motion  Center  of  mass  Centrifugal            “Force”  Coriolis  “Force”.    

presentation  to  bring  

gravitational  theories  and  laws  picture’s  closer  to  

students.    Spend  at  most  20  minutes.  Use  School  Adaptation,  video  tape  to  illustrate  

“Kepler’s  Laws,  Navigating  in  Space,  and  The  Apple  and  The  

Moon.”  www.discoveryschools.com  

 

Plato  student  

Interactive  

program,  

[email protected]  

Error!  Hyperlink  reference  not  

valid.    

Movie  segments  can  also  from:  

Page 13: Physics%–%1st%Quarter% - Lorain City School District · PDF file11 L12text’ ... Solving’ Many’ problems ... predict’the’effects’of’forces’onmore’complex’systems’of’objects’and’to’analyze’objects

Lorain  City  School  District  Scope,  Sequence  and  Pacing  Guides  

Physics    Grade  12    

13    Common  Formative  Assessments  will  be  implemented  daily.  

 

“Annehttp://www.learner.org/resources/series42.html?pop=yes&pid=582”  

 https://ieeetv.ieee.org/channels/pels?utm_source=google&utm_medium=cpc&utm_content=Electromagnetism&utm_campaign=IE

EEtv+-­‐+Channels+-­‐

+Power+Electronics    

Content  Elaborations:    Gravitational  Potential  Energy:    When  two  attracting  masses  interact,  the  kinetic  energies  of  both  objects  change  but  neither  is  acting  as  the  energy  source  or  the  receiver.  Instead,  the  energy  is  transferred  into  or  out  of  the  gravitational  field  around  the  system  as  gravitational  potential  energy.  A  single  mass  does  not  have  gravitational  potential  energy.  Only  the  system  of  attracting  masses  can  have  gravitational  potential  energy.  When  two  masses  are  moved  farther  apart,  energy  is  transferred  into  the  field  as  gravitational  potential  energy.  When  two  masses  are  moved  closer  together,  gravitational  potential  energy  is  transferred  out  of  the  field.    

Additional  Resources  or  “Examples:”  

• the  inner  lane  of  a  track  has  a  radius  of  20  meter  and  the  outer  lane  has  a  radius  of  30  meter.  Compare  the  centripetal  acceleration  for  two  runners  in  these  two  lanes  that  both  have  constant  speeds  of  10  m/s.  Is  the  centripetal  acceleration  less  than  g  (if  not,  there  probably  isn't  enough  static  friction  to  keep  the  runners  on  a  circular  path  and  their  feet  will  start  to  slip).  If  the  runners  are  traveling  counterclockwise,  what  is  the  direction  of  the  acceleration?  Centripetal  acceleration  of  each  is:    

Page 14: Physics%–%1st%Quarter% - Lorain City School District · PDF file11 L12text’ ... Solving’ Many’ problems ... predict’the’effects’of’forces’onmore’complex’systems’of’objects’and’to’analyze’objects

Lorain  City  School  District  Scope,  Sequence  and  Pacing  Guides  

Physics    Grade  12    

14    Common  Formative  Assessments  will  be  implemented  daily.  

 

r=20m:  ac  =  Vc2/r  =  (10  m/s)2/(20  m)  =  5  m/s2  r=30m:  ac  =  Vc2/r  =  (10  m/s)2/(30  m)  =  3.3  m/s2  

The  acceleration  on  the  inside  lane  is  significantly  bigger  than  for  the  outside,  but  both  are  well  under  g  in  magnitude.  The  acceleration  points  to  the  center  of  the  curve,  which  will  be  to  each  runner's  left.    

• A  golf  club  has  a  speed  just  before  striking  the  ball  of  40  m/s,  if  the  club  head  is  1.5  meter  from  the  axis  of  rotation,  what  is  the  magnitude  and  direction  centripetal  acceleration?  If  the  club  has  a  mass  of  0.2  kg  (weight  is  7  oz.),  what  net  force  is  needed  to  produce  this  acceleration?  centripetal  acceleration:  just  use  the  formula:    

ac  =  Vc2/r  =  (40  m/s)2/(1.5  m)  =  1067  m/2  

The  direction  is  toward  the  center  of  rotation  (between  the  golfer's  shoulders).  

Force:  The  net  force  is  always  given  by  Newton's  Second  Law:  

F  =  m*a  =  (0.20  kg)*(1067  m/2)  =  213  N  (about  45  lb)  

Note  that  this  force  (plus  the  weight  of  the  club)  is  provided  by  the  golfer's  hands  on  the  club  (static  friction).  Since  the  maximum  value  of  S  is  related  to  the  normal  force  strength,  the  golfer  must  hold  onto  the  club  grip  tightly  enough  [for  most  people  this  is  no  problem  with  two  hands].    

 

• The  most  extreme  example  of  centripetal  acceleration  in  sport:  A  hammer  (a  16  lb  weight,  7.27  kg)  on  a  1.2  meter  wire  with  a  handle)  can  be  thrown  85  meter.  To  achieve  that  distance  at  the  ideal  launch  angle,  the  launch  speed  must  be  at  least    

R  =  Vc2/g  Vc2  =  (D*g)  =  840  (m/s)2    

Vc  =29  m/s  

The  radius  at  release  is  about  1.0  meter  (because  the  thrower  is  leaning  back).  What  was  the  centripetal  acceleration  just  at  release?    

Page 15: Physics%–%1st%Quarter% - Lorain City School District · PDF file11 L12text’ ... Solving’ Many’ problems ... predict’the’effects’of’forces’onmore’complex’systems’of’objects’and’to’analyze’objects

Lorain  City  School  District  Scope,  Sequence  and  Pacing  Guides  

Physics    Grade  12    

15    Common  Formative  Assessments  will  be  implemented  daily.  

 

a  =  Vc2/r  =  840  (m/s)2/(1.0  m)  =  840  m/s2  =  85*  g!  

The  force  at  release  is  tremendous:    

F  =  m*a  =  (7.27  kg)*(840  m/s2)  =  6107  N  (=  1360  lb!)  

The  thrower  must  exert  this  force  along  the  cable  (which  the  cable  exerts  back  on  the  thrower).  There  is  no  way  the  thrower  could  resist  this  force  using  only  friction  (so  the  thrower  cannot  be  stationary):  the  thrower  at  the  other  end  of  the  wire  is  also  rotating  and  the  reaction  force  of  the  hammer  pulling  the  wire  provides  the  necessary  force  for  the  thrower's  centripetal  acceleration.    

 

End  Early  1st  Quarter  (9  Weeks)  District  

Short  Cycle  Assessment  

Strand   Topic   Content  Statement   Days  Clear  

Learning  Target  

Vocabulary   Core  Resource  

Additional  Resources   Assessment  

    In  regard  to  launching  your  own  rocket,  you  may  assign  points  as  you  wish,  as  long  as  you  write  “a  rubric”  that  would  be  worth  150  points.    Also,  make  sure  you  assign  a  day  to  launch  that  weather  is  a  factor.      

           

   

Content  Elaborations:    1. Instructional  Strategies  and  Resources    

This  section  provides  additional  support  and  information  for  educators.  These  are  strategies  for  actively  engaging  students  with  the  topic  and  for  providing  hands-­‐on,  minds-­‐on  observation  and  exploration  of  the  topic,  including  authentic  data  resources  for  scientific  inquiry,  experimentation  and  problem-­‐based  tasks  that  incorporate  technology  and  technological  and  engineering  design.  Resources  selected  are  printed  or  Web-­‐based  materials  that  directly  relate  to  the  particular  Content  Statement.  It  is  not  intended  to  be  a  prescriptive  list  of  lessons.    •  “Collision  Lab”:  is  an  interactive  simulation  that  allows  students  to  Investigate  collisions  on  an  air  hockey  table.  Students  can  vary  the  number  of  discs,  masses,  elasticity  and  initial  conditions  to  see  if  momentum  and  kinetic  energy  are  conserved.    •  “Forces  and  Motion”  :  is  an  interactive  simulation  that  allows  students  to  explore  the  forces  present  when  a  filing  cabinet  is  pushed.  Students  can  create  

Page 16: Physics%–%1st%Quarter% - Lorain City School District · PDF file11 L12text’ ... Solving’ Many’ problems ... predict’the’effects’of’forces’onmore’complex’systems’of’objects’and’to’analyze’objects

Lorain  City  School  District  Scope,  Sequence  and  Pacing  Guides  

Physics    Grade  12    

16    Common  Formative  Assessments  will  be  implemented  daily.  

 

an  applied  force  and  see  the  resulting  friction  force  and  total  force  acting  on  the  cabinet.  Graphs  show  forces  vs.  time,  position  vs.  time,  velocity  vs.  time,  and  acceleration  vs.  time.  A  force  diagram  of  all  the  forces  (including  gravitational  and  normal  forces)  is  shown.  

2. http://sciencefair.math.iit.edu/projects/pisa/  3. https://ieeetv.ieee.org/channels/pels?utm_source=google&utm_medium=cpc&utm_content=Electromagnetism&utm_campaign=IEEEtv+-­‐

+Channels+-­‐+Power+Electronics  Have  you  ever  wanted  to  launch  your  own  rocket?”  This  project  is  worth  150  points.    The  materials  needed  are  easy  to  obtain.    The  procedure  is  also  easily  manipulated  and  simple  to  follow.    The  math  is  calculus  based,  and  it  is  fun.  Follow  the  URL  address  as  the  following:  http://www.scienceinschool.org/repository/docs/issue22_rockets.pdf    

Strand   Topic   Content  Statement   Days   Clear  Learning  Target   Vocabulary   Core  Resource   Additional  

Resources   Assessment  

Forces,  Momentum  and  Energy  

Chapters  9:  Momentum  and  Its  

Conservation,  

1.  Impulse  and  Momentum  

2.  Conservation  of  Momentum  

 

Chapter  10:  Energy,  Work,  and  Simple  Machines  

1.  Energy  and  Work  

WHST.11-­‐12.1.  Write  arrangements  focused  on  Discipline-­‐specific  content.  

 WHST.11-­‐12.2.  

Write  informative/explanatory  texts,  including  the  narration  of  historical  events,  scientific  

procedures/experiments,  or  technical  processes.  

   WHST.11-­‐12.2.  Write  informative/explanatory  texts,  including  the  narration  of  historical  events,  scientific  procedures/experiments,  or  technical  processes.    (Note:  Teachers  should  be  

43   Chapter  9:  Momentum  and  

Its  Conservation.  

1.    I  can  determine  the  momentum  of  different  processes  as  illustarted  in  the  examles  below:  

a.  60-­‐kg  halfback  moving  eastward  at  9  m/s.  ANS:  p=m*v=  540  Kg*m/s,  East  

b.  1000-­‐kg  car  

Chapter  9:  Impulse  Momentum  Impulse-­‐Momentum            theorem  Angular    momentum  Angular            impulse-­‐          angular  momentum            theorem  Closed            system  Isolated            system    Law  of    conservation    of    momentum    Law  of  

Students  need  to  perform  “PHYSICS  LAB.”  As  

suggested  on  p.p.  302-­‐303.  

 Use  this  lab  as  an  alternative  “Inquiry  Lab.”  As  suggested  on  p.p.  303  in  textbook.  

 Students  must    know  more  about  energy  by  referring  to  the  internet,  and  log  on  to  “physics  Online-­‐through  

Physicspp.com

www.discoveryschools.co

m    

www.physicspp.com/vocabulary_puzzle

maker  http://wufs.wustl.edu/lap

is2/    

Plato  student  interactive  program,  “Work,  Power,  

Energy  and  its  

conservation.”  

Use  the  conclusion  questions  from  any  minilab  and  Labs  as  a  review  session  to  figure  the  students’  understandings  as  well  as  achievements    Re-­‐teach  by  giving  additional  problems  as  suggested  in  the  back  of  each  chapter  (9-­‐11).    When  the  students  enter  

Page 17: Physics%–%1st%Quarter% - Lorain City School District · PDF file11 L12text’ ... Solving’ Many’ problems ... predict’the’effects’of’forces’onmore’complex’systems’of’objects’and’to’analyze’objects

Lorain  City  School  District  Scope,  Sequence  and  Pacing  Guides  

Physics    Grade  12    

17    Common  Formative  Assessments  will  be  implemented  daily.  

 

2.  Machines  

Chapter  11:    Energy  and  Its  Conservation  

1.  The  Many  Forms  of  energy.  

2.  Conservation  of  Energy  

aware  that  the  common  reaction  classifications  

that  are  often  used  in  high  school  physics  courses  

often  lead  to  misconceptions  because  they  are  not  based  on  the  actual  physics,  but  on  

surface  features  that  may  happen  when  two  

substances  are  mixed  may  be  similar  from  one  

system  to  another  (e.g.,  exchanging  partners),  even  though  the  

underlying  physics  is  not  the  same.  However,  they  may  be  useful  in  making  predictions  about  what  )    

moving  northward  at  20  m/s.  ANS:  p=m*v=  20,000  Kg*m/s  

c.  40-­‐kg  freshman  moving  southward  at  2  m/s.    ANS:  p=  m*v=  80  Kg*m/s  

2.  I  can  recognize  the  appropriateness  and  value  of  basic  questions  “What  had  happened?”  “What  are  the  odds?”  and  “How  do  scientists  and  engineers  know  what  will  happen?”  

3.  I  can  explain  how  an  object’s  kinetic  energy  depends  on  its  mass  and  its  

conservation  of  angular  momentum    Chapter  10:  Work  Energy  Kinetic            energy  Work-­‐energy          theorem  Joule  Power  Watt  Machine  Effort  force  Resistance            force  Mechanical        advantage    Ideal  mechanical  advantage    Efficiency  Compound            machine    Chapter  11:  Rotational            kinetic              energy    

”  Also,  Energy  flow  diagrams  picture  energy  transformatio

ns  in  an  accurate,  

quantitative  and  

conceptually  transparent  manner.  

Energy  Flow  Diagrams  for  Teaching  Physics  

Concepts  is  a  paper  that  was  published  in  The  Physics  Teacher  that  outlines  how  to  use  these  tools  in  the  classroom.  It  proceeds  from  

simple  processes  to  complex  socially  significant  processes  

such  as  global  warming.  

 

“The  Mechanical  Universe,  high  school  adaptation,  video  tape,  “Conservation  of  Energy”  

 Distant  Learning  

Conferences.    

More  movie  segments  can  be  observed  

from:  “Annehttp://www.learner.org/resources/series42.html?pop=yes&pid=582”  

 Read  and  answer  questions  

proposed  for  “Technology  and  Society-­‐

Going  Further”  on  p.p.  304  in  

text.    

classroom,  the  teacher  needs  to  post  a  similar  problem  (either  suggested  as  a  review  sessions  or  in  the  back  of  each  chapter  9-­‐11)  to  students  have  been  learning  the  concept  as  a  guided  practice  and  reassures  students  that  reviews  are  always  available.  

 Use  study  guides  such  as  the  ones  appear  on  p.p.  305  in  text.    Review  packets  (chapters  9,  10  and  11)  Open  Book  Quizzes  (9,  10,  

Page 18: Physics%–%1st%Quarter% - Lorain City School District · PDF file11 L12text’ ... Solving’ Many’ problems ... predict’the’effects’of’forces’onmore’complex’systems’of’objects’and’to’analyze’objects

Lorain  City  School  District  Scope,  Sequence  and  Pacing  Guides  

Physics    Grade  12    

18    Common  Formative  Assessments  will  be  implemented  daily.  

 

directional  speed  (KE  =  ½  mν2).  

4.  I  can  

demonstrate  that  

near  Earth’s  surface  an  object’s  gravitational  potential  energy  depends  upon  its  weight  (m*g*h)  where  m  is  the  object’s  mass  and  g  is  acceleration  due  to  gravity)  and  height  (h)  above  reference  surface  (PE  =  mgh)  

Chapter  10:  Energy,  Work,  and  Simple  Machines  

Chapter  11:  

Gravitational            potential            energy    Reference            level    Elastic            potential            energy    Law  of  conservation  of  energy    Mechanical            energy    Thermal            energy  Elastic            collision  Inelastic            collision                    

Assessment  Problems  Teacher  

Work  Book-­‐Chapter  11  

 Additional  Labs.  Such  as  the  one  

suggested  in  text  book-­‐

“Launch  Lab”  on  p.p.  285.  

 https://ieeetv.ieee.org/channels/pels?utm_source=google&utm_medium=cpc&utm_content=Electromagnetism&utm_campaign=IE

EEtv+-­‐+Channels+-­‐+Power+Elec

tronics  

and  11)  Exams  (9,  10,  and  11)  

Page 19: Physics%–%1st%Quarter% - Lorain City School District · PDF file11 L12text’ ... Solving’ Many’ problems ... predict’the’effects’of’forces’onmore’complex’systems’of’objects’and’to’analyze’objects

Lorain  City  School  District  Scope,  Sequence  and  Pacing  Guides  

Physics    Grade  12    

19    Common  Formative  Assessments  will  be  implemented  daily.  

 

Energy  and  Its  Conservation.  

1.  I  can  Follow  up  the  discussion  on  conservation  of  energy  by  using  “connecting  Math  to  Physics”  as  illustrated  on  p.p.  295  in  textbook.              2.  I  can  use  the  Example  on  Problem  #  2,  as  proposed  on  p.p.  296;  after  I  make    sure  I  go  over  the  analysis  and  solution  (Step-­‐by-­‐Step).        3.  Also,  I  can  use  the  Math  Handbook,  “Square  and  Cube  Roots,  p.p.  839-­‐840  to  assess  and  evaluate  my  

Page 20: Physics%–%1st%Quarter% - Lorain City School District · PDF file11 L12text’ ... Solving’ Many’ problems ... predict’the’effects’of’forces’onmore’complex’systems’of’objects’and’to’analyze’objects

Lorain  City  School  District  Scope,  Sequence  and  Pacing  Guides  

Physics    Grade  12    

20    Common  Formative  Assessments  will  be  implemented  daily.  

 

understanding.    

4.  I  can  learn  what  was  explained  in  step  26  above,  in  regard  to  Example  Problem  #  3,  p.p.  299.  

5.  I  can  discuss  the  suggested  misconception  on  p.p.  298,  as  well  as  using  models.    I  can  also  refer  to  the  diagrams  illustrated  in  Figure  11-­‐4  on  p.p.  298  to  represent  the  three  types  of  collisions.  

6.  As  a  reinforcement  of  my  understanding  to  collision  and  

Page 21: Physics%–%1st%Quarter% - Lorain City School District · PDF file11 L12text’ ... Solving’ Many’ problems ... predict’the’effects’of’forces’onmore’complex’systems’of’objects’and’to’analyze’objects

Lorain  City  School  District  Scope,  Sequence  and  Pacing  Guides  

Physics    Grade  12    

21    Common  Formative  Assessments  will  be  implemented  daily.  

 

its  elasticity,  I  can  use  what  is  suggested  on  p.p.  299.  

 Content  Elaborations: Energy    In  physical  science,  the  role  of  strong  nuclear  forces  in  radioactive  decay,  half-­‐lives,  fission  and  fusion,  and  mathematical  problem  solving  involving  kinetic  energy,  gravitational  potential  energy,  energy  conservation  and  work  (when  the  force  and  displacement  were  in  the  same  direction)  were  introduced.  In  this  course,  the  concept  of  gravitational  potential  energy  is  understood  from  the  perspective  of  a  field,  elastic  potential  energy  is  introduced  and  quantified,  nuclear  processes  are  explored  further,  the  concept  of  mass-­‐energy  equivalence  is  introduced,  the  concept  of  work  is  expanded,  power  is  introduced,  and  the  principle  of  conservation  of  energy  is  applied  to  increasingly  complex  situations.    Energy  must  be  explored  by  analyzing  data  gathered  in  scientific  investigations.  Computers  and  probes  can  be  used  to  collect  and  analyze  data. Gravitational  Potential  Energy    When  two  attracting  masses  interact,  the  kinetic  energies  of  both  objects  change  but  neither  is  acting  as  the  energy  source  or  the  receiver.  Instead,  the  energy  is  transferred  into  or  out  of  the  gravitational  field  around  the  system  as  gravitational  potential  energy.  A  single  mass  does  not  have  gravitational  potential  energy.  Only  the  system  of  attracting  masses  can  have  gravitational  potential  energy.  When  two  masses  are  moved  farther  apart,  energy  is  transferred  into  the  field  as  gravitational  potential  energy.  When  two  masses  are  moved  closer  together,  gravitational  potential  energy  is  transferred  out  of  the  field.    Energy  in  Springs    The  approximation  for  the  change  in  the  potential  elastic  energy  of  an  elastic  object  (e.g.,  a  2spring)  is  ΔE  elastic  =  ½  k  Δx,  where  Δx  is  the  distance  the  elastic  object  is  stretched  or  Compressed  from  its  relaxed  length.  Work  and  Power    Work  can  be  calculated  for  situations  in  which  the  force  and  the  displacement  are  at  angles  to  one  another  using  the  equation  W  =  F  Δx  (cos  θ)  where  W  is  the  work,  F  is  the  force,  Δx  is  the  displacement,  and  θ  is  the  angle  between  the  force  and  the  displacement.  This  means  when  the  force  and  the  displacement  are  at  right  angles,  no  work  is  done  and  no  energy  is  transferred  between  the  objects.  Such  is  the  case  for  circular  motion.  The  rate  of  energy  change  or  transfer  is  called  power  (P)  and  can  be  mathematically  represented  by  P  =  ΔE  /  Δt  or  P  =  W  /  Δt.  Power  is  a  scalar  property.  The  unit  of  power  is  the  watt  (W),  which  is  equivalent  to  one  Joule  of energy transferred  in  one  second  (J/s).      

Page 22: Physics%–%1st%Quarter% - Lorain City School District · PDF file11 L12text’ ... Solving’ Many’ problems ... predict’the’effects’of’forces’onmore’complex’systems’of’objects’and’to’analyze’objects

Lorain  City  School  District  Scope,  Sequence  and  Pacing  Guides  

Physics    Grade  12    

22    Common  Formative  Assessments  will  be  implemented  daily.  

 

 

 

Page 23: Physics%–%1st%Quarter% - Lorain City School District · PDF file11 L12text’ ... Solving’ Many’ problems ... predict’the’effects’of’forces’onmore’complex’systems’of’objects’and’to’analyze’objects

Chapters  1-­‐>3:

Name ___________________________________________Period _________________

Chapter 1 Test

MULTIPLE CHOICE Identify the letter of the choice that best completes the statement or answers the question. ____1. 1 micrometer (1 µm) equals: a. 10–3 m c. 103 m b. 10–6 m d. 106 m ____2. According to Boyles’ law, PV = constant. If a graph is plotted with the pressure P against the volume V, the graph would be a(n): a. straight line c. hyperbola b. parabola d. ellipse

____3. The time period T of a simple pendulum is given by the relation , where L is

the length of the pendulum and g is the acceleration due to gravity at the place. From the above relation, we can say that:

a. T ∝ g c. T2 ∝ g b. T ∝ L d. T2 ∝ L

____4. The slope of the graph given below is:

a.

b.

c.

d.

Page 24: Physics%–%1st%Quarter% - Lorain City School District · PDF file11 L12text’ ... Solving’ Many’ problems ... predict’the’effects’of’forces’onmore’complex’systems’of’objects’and’to’analyze’objects

COMPLETE EACH SENTENCE OR STATEMENT: 5. The method of treating the units as algebraic quantities that can be canceled is called ____________________ analysis. 6. The valid digits in a measurement are called ____________________. 7. When the graph of best fit is a straight line, the dependent and independent variables have a(n) ____________________ relationship. 8. The acceleration of the body in the graph given below is ____________________.

SHORT ANSWER ESSAY: 9. Two objects have masses m1 and m2 and their centers are separated by a distance d. The force

of gravitation between these objects is given by the expression .

Here G is the gravitational constant. Find the units of measurement of G. 10. Write the number 13900 using scientific notation. What is the number of significant digits in this number?

Page 25: Physics%–%1st%Quarter% - Lorain City School District · PDF file11 L12text’ ... Solving’ Many’ problems ... predict’the’effects’of’forces’onmore’complex’systems’of’objects’and’to’analyze’objects

11. What is two-point calibration? 12. The position-time graphs of two runners, Ian and Nick, are shown below. How can the faster runner be determined using these graphs?

PROBLEM 13. A body has a velocity of 72 km/hr. Find its value in m/s. 14. The velocity of sound in air is 332 m/s. If the unit of length is km and the unit of time is hour, what is the value of velocity? 15. How many seconds are there in the month of February in a leap year? ESSAY 16. Analyze the graph and answer the following questions.

Ian

Nick

Posi

tion

(m)

Time (s)1 2 3 4 5 6 7 8 9 t

1

2

3

4

5

6

7

8

9

10d

Page 26: Physics%–%1st%Quarter% - Lorain City School District · PDF file11 L12text’ ... Solving’ Many’ problems ... predict’the’effects’of’forces’onmore’complex’systems’of’objects’and’to’analyze’objects

a. Name the independent and dependent variables in the graph. b. Name the type of relationship between the variables. c. Find the speed of the body. 17. An object has an initial velocity v and uniform acceleration a. If it covers a distance d, then its final velocity, v , is given by the expression, . a. Does this equation give the correct unit of measurement of vf?

b. Is this a base unit or a derived unit?

c. Rewrite this equation to find the distance traveled in terms of initial velocity, final velocity,

and acceleration.

18. Consider the expression for the arithmetic operation, 0.652 km + 195 cm + 1800 cm.

a. Is any conversion required to do this operation?

b. Convert all the measurements to meters and then solve this expression.

c. How many significant digits are present in the obtained answer?

19. Jessica is using a micrometer screw (screw gauge) to measure the diameter of a wire. While

recording the measurements, she views the readings from different angles.

a. Is there any difference in the recorded values of the same measurement reading, when viewed

from different angles?

b. Name the phenomenon that causes this difference.

c. Does this affect the precision of a measurement made by the student?

Page 27: Physics%–%1st%Quarter% - Lorain City School District · PDF file11 L12text’ ... Solving’ Many’ problems ... predict’the’effects’of’forces’onmore’complex’systems’of’objects’and’to’analyze’objects

20. While performing an experiment to find the value of acceleration due to gravity, g, Lee made

observations for the time period of a simple pendulum. He only changed the length of the

pendulum and recorded different values of time period as given in the table.

Length L of pendulum in cm

25 36 49 64 81 100

Time for one oscillation Time period T in s

1.005 1.205 1.405 1.610 1.810 2.01

a. Is there any linear relationship between the values of length and time period of the simple pendulum? b. The L-T graph is a straight line passing through the origin. Find the relationship between the length and time period of the pendulum. c. What are the units of the constant obtained as a ratio of L and T ? CHAPTER 1ANSWER KEY: MULTIPLE CHOICE 1. ANS: B 2. ANS: C 3. ANS: D 4. ANS: B COMPLETION 5. ANS: dimensional 6. ANS: significant digits 7. ANS: linear 8. ANS:

Page 28: Physics%–%1st%Quarter% - Lorain City School District · PDF file11 L12text’ ... Solving’ Many’ problems ... predict’the’effects’of’forces’onmore’complex’systems’of’objects’and’to’analyze’objects

SHORT ANSWER

9. ANS:

10. ANS: Using scientific notation, the number 13900 can be expressed as 1.39 × 104. This number has three significant digits. 11. ANS: Two-point calibration is the method of checking the accuracy of an instrument. In this method, an instrument is checked for zero error and for the correctness of a reading, while measuring an accepted standard. 12. ANS: The position-time graphs of both runners show linear relationship between their positions and time. These straight line graphs show that the runners have uniform speeds. The slopes of these lines of graphs give the value of the speeds of the runners. The steeper the position-time graph of a runner, higher is his speed. Therefore, Ian is the faster runner. PROBLEM 13. ANS:

14. ANS:

15. ANS: 2505600 s ESSAY 16. ANS: a. Time is the independent variable and distance is the dependent variable. b. Linear relationship exists between the variables.

c.

17. ANS:

Page 29: Physics%–%1st%Quarter% - Lorain City School District · PDF file11 L12text’ ... Solving’ Many’ problems ... predict’the’effects’of’forces’onmore’complex’systems’of’objects’and’to’analyze’objects

a. To find the unit of measurement of v , substitute the units of d, a, and v . The unit of is m2/s2. Therefore, the unit of v is m/s, which is the correct unit for the measurement of velocity. b. The unit of measuring velocity is a derived unit. It is created using two basic SI units, meter and second.

c. The equation for finding distance is .

18. ANS: a. To solve any arithmetic operation involving physical quantities, all the quantities involved should have the measurement units of the same system of units. In this expression, 0.615 km should be first converted to cm, or magnitudes in cm should be converted to km. b. 0.652 km = 652 m, 195 cm = 1.95 m, 1800 cm = 18 m Therefore, 0.652 km + 195 cm + 1800 cm becomes 652 m + 1.95 m + 18 m = 672 m c. The number has three significant digits. 19. ANS: a. The recorded values of the same measurement reading differ when the reading is viewed from different angles. b. The phenomenon that causes this difference is called parallax. c. Parallax does not affect the precision of a measurement, but it affects the accuracy of the measurements. 20. ANS: a. The time period of a simple pendulum increases with length. However, from the readings given in the table, it is clear that T is not proportional to L. b. The straight line graph in case of L-T graph shows that L is directly proportional to T .

c. Since , therefore = constant. The units of this constant are m/s .

CHAPTER 1 ANSWER KEY WITH ANALYSIS MULTIPLE CHOICE: 1. ANS: B 10–6 is equivalent to the metric prefix micro (µ). Therefore, 1 micrometer (1 µm) = 10–6 m. DIF: 2 REF: p.p. 6 OBJ: 1.1.2 Use the metric system. TOP: Use the metric system. KEY: SI units MSC: 1 NOT: /a/ The metric prefix milli (m) is equivalent to 10e–3 m. /b/ Correct! /c/ The metric prefix kilo (k) is equivalent to 10e+3 m. /d/ The metric prefix mega (M) is equivalent to 10e+6 m.

Page 30: Physics%–%1st%Quarter% - Lorain City School District · PDF file11 L12text’ ... Solving’ Many’ problems ... predict’the’effects’of’forces’onmore’complex’systems’of’objects’and’to’analyze’objects

2. ANS: C The pressure P is inversely proportional to the volume V. For inverse relation, the graph between the variables is a hyperbola. DIF: 2 REF: p.p. 18 OBJ: 1.3.3 Recognize common relationships in graphs. STO: 12.D.3.2 TOP: Recognize common relationships in graphs. KEY: Inverse relationship MSC: 3 NOT: /a/ P and V do not have a linear relationship. /b/ P and V do not have a quadratic relationship. /c/ Correct! /d/ P and V have an inverse relationship, hence the graph is a hyperbola. 3. ANS: D

. On squaring both sides we get, .

Therefore, T2 ∝ L. DIF: 2 REF: p.p. 17 OBJ: 1.3.3 Recognize common relationships in graphs. STO: 12.D.3.2 TOP: Recognize common relationships in graphs. KEY: Linear relationship MSC: 3 NOT: /a/ T is inversely proportional to the square root of g. /b/ T is directly proportional to the square root of L. /c/ The square of T is inversely proportional to g. /d/ Correct! 4. ANS: B The slope is the ratio of the vertical change to the horizontal change. DIF: 1 REF: p.p. 17 OBJ: 1.3.1 Graph the relationship between independent and dependent variables. TOP: Graph the relationship between independent and dependent variables. KEY: Slope MSC: 2 NOT: /a/ Reciprocate the ratio. /b/ Correct! /c/ PR is neither the vertical nor the horizontal change. /d/ PR is the horizontal change and not the vertical change. COMPLETION 5. ANS: dimensional DIF: 1 REF: p.p. 6 OBJ: 1.1.3 Evaluate answers using dimensional analysis. TOP: Evaluate answers using dimensional analysis. KEY: Dimensional analysis MSC: 1 6. ANS: significant digits DIF: 1 REF: p.p. 7 OBJ: 1.1.4 Perform arithmetic operations using scientific notation. TOP: Perform arithmetic operations using scientific notation. KEY: Significant digits MSC: 1

Page 31: Physics%–%1st%Quarter% - Lorain City School District · PDF file11 L12text’ ... Solving’ Many’ problems ... predict’the’effects’of’forces’onmore’complex’systems’of’objects’and’to’analyze’objects

7. ANS: linear DIF: 1 REF: p.p. 16 OBJ: 1.3.3 Recognize common relationships in graphs. STO: 12.D.3.2 TOP: Recognize common relationships in graphs. KEY: Linear relationship MSC: 2 8. ANS: DIF: 2 REF: p.p. 17 OBJ: 1.3.1 Graph the relationship between independent and dependent variables. TOP: Graph the relationship between independent and dependent variables. KEY: Slope MSC: 2 SHORT ANSWER 9. ANS:

DIF: 2 REF: p.p. 6 OBJ: 1.1.3 Evaluate answers using dimensional analysis. TOP: Evaluate answers using dimensional analysis. KEY: Dimensional analysis MSC: 2 10. ANS: Using scientific notation, the number 13900 can be expressed as 1.39 × 104. This number has three significant digits. DIF: 2 REF: p.p. 7 OBJ: 1.1.4 Perform arithmetic operations using scientific notation. TOP: Perform arithmetic operations using scientific notation. KEY: Significant digits MSC: 2 11. ANS: Two-point calibration is the method of checking the accuracy of an instrument. In this method, an instrument is checked for zero error and for the correctness of a reading, while measuring an accepted standard. DIF: 2 REF: p.p. 13 OBJ: 1.2.2 Determine the precision of measured quantities. STO: 9.D.D.4 TOP: Determine the precision of measured quantities.

Page 32: Physics%–%1st%Quarter% - Lorain City School District · PDF file11 L12text’ ... Solving’ Many’ problems ... predict’the’effects’of’forces’onmore’complex’systems’of’objects’and’to’analyze’objects

KEY: Accuracy MSC: 2 12. ANS: The position-time graphs of both runners show linear relationship between their positions and time. These straight line graphs show that the runners have uniform speeds. The slopes of these lines of graphs give the value of the speeds of the runners. The steeper the position-time graph of a runner, higher is his speed. Therefore, Ian is the faster runner. DIF: 2 REF: p.p. 16 OBJ: 1.3.2 Interpret graphs. TOP: Interpret graphs. KEY: Linear relationship MSC: 2 PROBLEM 13. ANS:

DIF: 2 REF: p.p. 6 OBJ: 1.1.3 Evaluate answers using dimensional analysis. TOP: Evaluate answers using dimensional analysis. KEY: Dimensional analysis MSC: 2 NOT: 1 km = 1000 m and 1 hr = 60*60 s = 3600 s 14. ANS:

DIF: 3 REF: p.p. 7 OBJ: 1.1.3 Evaluate answers using dimensional analysis. TOP: Evaluate answers using dimensional analysis. KEY: Dimensional analysis MSC: 2 NOT: 1 m =0.001 km and 1 s = 1/3600 hr 15. ANS: 2505600 s DIF: 3 REF: p.p. 6 OBJ: 1.1.3 Evaluate answers using dimensional analysis. TOP: Evaluate answers using dimensional analysis. KEY: Dimensional analysis MSC: 2 NOT: In a leap year, in the month of February, there are 29 days. Therefore, the number of seconds = 29*24*60*60 = 2505600 s.

Page 33: Physics%–%1st%Quarter% - Lorain City School District · PDF file11 L12text’ ... Solving’ Many’ problems ... predict’the’effects’of’forces’onmore’complex’systems’of’objects’and’to’analyze’objects

ESSAY 16. ANS: a. Time is the independent variable and distance is the dependent variable. b. Linear relationship exists between the variables.

c.

DIF: 3 REF: p.p. 16 OBJ: 1.3.1 Graph the relationship between independent and dependent variables. TOP: Graph the relationship between independent and dependent variables. KEY: Independent and dependent variables MSC: 3 17. ANS: a. To find the unit of measurement of v , substitute the units of d, a, and v . The unit of is m2/s2. Therefore, the unit of v is m/s, which is the correct unit for the measurement of velocity. b. The unit of measuring velocity is a derived unit. It is created using two basic SI units, meter and second.

c. The equation for finding distance is .

DIF: 3 REF: p.p. 6, p.p. 7 OBJ: 1.1.3 Evaluate answers using dimensional analysis. TOP: Evaluate answers using dimensional analysis. KEY: Dimensional analysis MSC: 2 18. ANS: a. To solve any arithmetic operation involving physical quantities, all the quantities involved should have the measurement units of the same system of units. In this expression, 0.615 km should be first converted to cm, or magnitudes in cm should be converted to km. b. 0.652 km = 652 m, 195 cm = 1.95 m, 1800 cm = 18 m Therefore, 0.652 km + 195 cm + 1800 cm becomes 652 m + 1.95 m + 18 m = 672 m c. The number has three significant digits. DIF: 3 REF: p.p. 6 OBJ: 1.1.4 Perform arithmetic operations using scientific notation. TOP: Perform arithmetic operations using scientific notation. KEY: Significant digits MSC: 1 19. ANS: a. The recorded values of the same measurement reading differ when the reading is viewed from different angles. b. The phenomenon that causes this difference is called parallax.

Page 34: Physics%–%1st%Quarter% - Lorain City School District · PDF file11 L12text’ ... Solving’ Many’ problems ... predict’the’effects’of’forces’onmore’complex’systems’of’objects’and’to’analyze’objects

c. Parallax does not affect the precision of a measurement, but it affects the accuracy of the measurements. DIF: 3 REF: p.p. 13 OBJ: 1.2.2 Determine the precision of measured quantities. STO: 9.D.D.4 TOP: Determine the precision of measured quantities. KEY: Accuracy, Precision MSC: 3 20. ANS: a. The time period of a simple pendulum increases with length. However, from the readings given in the table, it is clear that T is not proportional to L. b. The straight line graph in case of L-T graph shows that L is directly proportional to T .

c. Since , therefore = constant. The units of this constant are m/s .

DIF: 3 REF: p.p. 16, p.p. 17 OBJ: 1.3.3 Recognize common relationships in graphs. STO: 12.D.3.2 TOP: Recognize common relationships in graphs. KEY: Linear relationship, Nonlinear relationship MSC: 3

Page 35: Physics%–%1st%Quarter% - Lorain City School District · PDF file11 L12text’ ... Solving’ Many’ problems ... predict’the’effects’of’forces’onmore’complex’systems’of’objects’and’to’analyze’objects

Name ___________________________________________________ Period _______________

Chapter 2 Test MULTIPLE CHOICE Identify the letter of the choice that best completes the statement or answers the question. ____1. A car is moving with a uniform speed of 15.0 m/s along a straight path. What is the distance covered by the car in 12.0 minutes? a. 1.02 × 10–3 km c. 8.00 × 10–5 km b. 1.80 × 10–1 km d. 1.08 × 101 km ____2. Which of the following is a pair of vector quantities? a. Speed — Distance c. Velocity — Displacement b. Velocity — Distance d. Speed — Displacement ____ 3. What is the resultant of two displacement vectors having the same direction? a. The resultant is the sum of the two displacements having the same direction as the

original vectors. b. The resultant is the difference of the two displacements having the same direction

as the original vectors. c. The resultant is the sum of the two displacements having the direction opposite to

the direction of the original vectors. d. The resultant is the sum of the two displacements having the direction

perpendicular to the direction of the original vectors. COMPLETION: Complete each sentence or statement. 4. A(n) ____________________ model is a simplified version of a motion diagram that represents the object in motion by a series of single points. 5. The vector that represents the sum of the other two vectors is called the ____________________. SHORT ANSWER: 6. What is a motion diagram? 7. What is the significance of defining a coordinate system to study the motion of an object? 8. How is displacement different from distance? Under what conditions can an object travel a certain distance and yet its resultant displacement be zero? 9. What is the equation of motion for average velocity?

Page 36: Physics%–%1st%Quarter% - Lorain City School District · PDF file11 L12text’ ... Solving’ Many’ problems ... predict’the’effects’of’forces’onmore’complex’systems’of’objects’and’to’analyze’objects

10. Define average velocity. 11. The position-time graph of an object is found to be a straight line passing through the origin. What information about the motion of the object is provided by the graph? 12. Given below is the position-time graph representing motions of two runners, Nick and Ian. Use this graph to determine which runner has greater average velocity.

13. Given below is the particle model of a boy skating on a smooth, pedestrian-free sidewalk. The time interval between successive dots is 2 s.

Plot a position-time graph to represent the motion of the boy.

Ian

Nick

Posi

tion

(m)

Time (s)1 2 3 4 5 6 7 8 9 10

1

2

3

4

5

6

7

8

9

10

Page 37: Physics%–%1st%Quarter% - Lorain City School District · PDF file11 L12text’ ... Solving’ Many’ problems ... predict’the’effects’of’forces’onmore’complex’systems’of’objects’and’to’analyze’objects

14. The position-time graph given below represents the motion of Ted returning home from the market on his bike. What is the similarity between his displacement and the average velocity?

15. Given below is the motion diagram of a small ball rolling straight on a frictionless surface.

The time interval between successive positions is 1 s.

a. What is the displacement of the ball after 3 s?

b. Where does the ball reach after 5 s?

c. After rolling for 7 s, the direction of motion of the ball is changed. It starts rolling toward

its starting point. Assume the coordinate system and the speed of the ball to remain unchanged.

What is the displacement of the ball between 7 s and 9 s?

Time (s)

Posi

tion

(m)

1 2 3 4 5 6 7 8 9 10 11 12 13–1

5

10

15

20

25

30

35

40

45

50

–5

–10

Page 38: Physics%–%1st%Quarter% - Lorain City School District · PDF file11 L12text’ ... Solving’ Many’ problems ... predict’the’effects’of’forces’onmore’complex’systems’of’objects’and’to’analyze’objects

16. Given below is the graph representing the position-time graphs of two swimmers (A and B), swimming in a pool along a straight line. Both the swimmers start from two different positions. Use the graph to find when and where swimmer B passes swimmer A.

17. The position-time graph of a pedestrian is given below. What is his displacement after 2.5 s?

18. What information is provided by the points on the line of a position-time graph of an object? 19. A boy starts from point A and moves 5 units toward the east, then turns back and moves 3 units toward the west. What is the displacement in the position of the boy? 20. What are the two attributes of the coordinate system chosen for a motion diagram? 21. How is the time interval affected when the origin of the coordinate system of a motion diagram is changed?

Swimmer A

Swimmer B

Posi

tion

(m)

Time (s)10 20 30 40 50 60 70 80

50

100

150

200

250

300

350

–50

–100

Posi

tion

(m)

Time (s)1 2 3 4 5 6 7 8 9 10

2

4

6

8

10

12

–2

–4

–6

–8

Page 39: Physics%–%1st%Quarter% - Lorain City School District · PDF file11 L12text’ ... Solving’ Many’ problems ... predict’the’effects’of’forces’onmore’complex’systems’of’objects’and’to’analyze’objects

22. What is the distance traveled by a vehicle in 12 minutes, if its speed is 35 km/h? 23. Given below is the position-time graph representing the motion of two friends, A and B, jogging in a park. Use this graph to find their displacements after 4 s.

ESSAY 24. A boy is cycling at a constant speed along a straight road to his school. The positions of the

boy after fixed time intervals are observed.

a. What type of position-time graph can be expected for the motion of the bicycle?

b. How can the average velocity of the bicycle be calculated?

c. How is the average speed of the bicycle related to its average velocity?

25. Linda starts for her school at eight in the morning. There is a straight road that connects her

house to the school. She reaches school after 3.5 minutes. Her physics teacher has asked her to

note her position after every 30 seconds on her way to school.

The values for her position (distance from her home) are listed with each time interval in the

table below:

Time (in seconds)

0 30 60 90 120 150 180 210

Position (in meters)

0 45 90 135 180 225 270 315

A

B

Posi

tion

(m)

Time (s)1 2 3 4 5 6 7 8 9 10

1

2

3

4

5

6

7

8

9

10

Page 40: Physics%–%1st%Quarter% - Lorain City School District · PDF file11 L12text’ ... Solving’ Many’ problems ... predict’the’effects’of’forces’onmore’complex’systems’of’objects’and’to’analyze’objects

Using this position time table, a. Draw a position-time graph to represent Linda’s motion from home to school.

b. Calculate the average velocity of Linda using the position-time graph.

c. What is the velocity of Linda on a return trip home if she reaches back home in the same

time of 3.5 minutes? Assume Linda’s house to be the fixed origin of the graph.

26. A mouse is running at a high speed between the kitchen and storeroom, situated along the

same line in a building. A boy observes the mouse picking up pieces of bread from the kitchen

and leaving them in a box in the storeroom. The boy observes the positions of the mouse after

fixed intervals. He finds that the mouse travels the same displacement of 2.0 m in every half

second. Starting from the door of the kitchen, the mouse reaches the door of the storeroom in 3.0

s.

a. Make a position-time table for the motion of the mouse from the kitchen to the

storeroom.

b. Draw the particle model of this motion.

c. Convert this model into the corresponding position-time graph.

27. Thomas cycles on a straight road near his house. After sometime, his sister Anna also starts

cycling. Their motions are represented by the position-time graphs given below.

a. How long had Thomas been cycling when Anna started cycling?

b. What is the position at which Anna crosses Thomas?

c. What is the difference between their positions after 5 s?

Page 41: Physics%–%1st%Quarter% - Lorain City School District · PDF file11 L12text’ ... Solving’ Many’ problems ... predict’the’effects’of’forces’onmore’complex’systems’of’objects’and’to’analyze’objects

28. Three railway stations, A, B, and C, are situated on a straight railway route. Station B is at 30

km in the east from station A and station C is at 60 km in east from station A. A train, with a

constant average velocity, starts from A and reaches B in 15 minutes, and reaches C in 30

minutes. Assuming that station A is the origin, find the following:

a. The position of the train moving from station A to station B after 10 minutes.

b. The position of the train moving from station A to station B after 10 minutes, if station B

is assumed to be the origin.

c. The position of the train moving from station A to station B after 10 minutes, if station C

is assumed to be the origin.

29. Tom drives his car at a constant speed on a straight road for two hours while going to meet

his friend. He noted the position of his car after every 15 minutes, assuming his house to be the

origin. On his way back home, he again noted the positions of the car after every 15 minutes,

without changing the origin. If on both of the trips, his car covers 25 km in every 15 minutes,

then:

a. Draw the position-time graphs of the car for the two sides of the journey.

b. Find the difference between the average velocities of the two sides of the journey.

c. Find what happens if during the return journey, the friend’s house is taken as the origin.

Page 42: Physics%–%1st%Quarter% - Lorain City School District · PDF file11 L12text’ ... Solving’ Many’ problems ... predict’the’effects’of’forces’onmore’complex’systems’of’objects’and’to’analyze’objects

CHAPTER 2 ANSWER KEY MULTIPLE CHOICE 1. ANS: D 2. ANS: C 3. ANS: A COMPLETION 4. ANS: particle 5. ANS: resultant SHORT ANSWER 6. ANS: Motion diagram is a series of images showing the positions of a moving object at equal time intervals. 7. ANS: A coordinate system tells about the location of the zero point of the variables defining the motion of the object to be studied. The coordinate system also explains the direction in which the values of the variables increase. 8. ANS: Displacement is a vector quantity, while distance is a scalar quantity. Displacement has both the magnitude and direction. Distance has only magnitude. The displacement of an object is equal to the difference between its final position and initial position. When an object starts moving from a certain point and after covering certain distance returns to its starting position, its displacement becomes zero. 9. ANS: The equation of motion for average velocity is . In this equation, d is the position of an object, is the average velocity of the object, t is the time and di is the initial position of the object. According to this equation, an object’s position is equal to the average velocity multiplied by time plus the initial position. 10. ANS: Average velocity of an object is defined as the change in position of the object moving along a straight line, divided by the time during which the change occurred. In symbolic form, the average velocity

Page 43: Physics%–%1st%Quarter% - Lorain City School District · PDF file11 L12text’ ... Solving’ Many’ problems ... predict’the’effects’of’forces’onmore’complex’systems’of’objects’and’to’analyze’objects

.

11. ANS: The straight line position-time graph of a moving object gives the information about the nature of the motion of the object. It is also used to calculate the average velocity of the object. Since the graph is a straight line, the object is undergoing uniform motion. The slope of this line gives the value of the average velocity of the object. 12. ANS: The average velocity of an object is equal to the slope of the line representing the position-time graph of the object. In this graph, the slope of Ian’s line is steeper than the slope of Nick’s line. A steeper slope indicates a greater change in displacement during each time interval. Average velocity for a given time interval is proportional to the change in the displacement. Therefore, the average velocity of Ian is more than the average velocity of Nick. 13. ANS: The position-time graph corresponding to this particle model is plotted below.

14. ANS: In this case, motion is in the negative direction. Therefore, displacement and velocity of the bike are negative. Hence, both the displacement and the velocity have a negative sign. 15. ANS: a. The value of displacement after 3 s is 12 m. b. After 5 s, the ball reaches at 20 m from its starting point. c. The position of the ball after 7 s = 28 m. The position of the ball after 9 s = 20 m. The direction of the motion of the ball is reversed after 7 s. Therefore, the displacement of the ball between 7 s and 9 s = Δd = 20 – 28 = –8 m. The negative sign shows that the direction of displacement in this case is negative, toward the starting point.

Time (s)

Posi

tion

(m)

2 4 6 8 10 12 14 16 18 20 22 24

5

10

15

20

25

30

35

40

45

50

Page 44: Physics%–%1st%Quarter% - Lorain City School District · PDF file11 L12text’ ... Solving’ Many’ problems ... predict’the’effects’of’forces’onmore’complex’systems’of’objects’and’to’analyze’objects

16. ANS: The point of intersection of the two graphs is 150 m at about 38 s. Swimmer B passes Swimmer A about 150 m beyond the origin 38 s after A has passed the origin. 17. ANS: The position-time graph of the pedestrian intersects the x-axis at 2.5 s. Thus, the displacement in 2.5 s is –10 m. 18. ANS: The points on the line of a position-time graph of an object represent the most likely positions of the moving object at the times between the recorded data points. 19. ANS:

The resultant displacement is 2 units toward the east, and his position is at 2. 20. ANS: The two attributes of the coordinate system chosen for a motion diagram are the origin and the axis of the coordinate system. 21. ANS: The time interval is a scalar quantity. Its value is not affected by the change in the relative position of the origin of the coordinate system. 22. ANS: Speed is given in km/h, therefore convert time in minutes to hours.

Distance traveled = (speed)(time taken) = (35 km/h)(12 min)(1 h/60 min) = 7.0 km 23. ANS: The displacements of A and B are the coordinates of the vertical line with their respective position-time graphs at 4 s.

Page 45: Physics%–%1st%Quarter% - Lorain City School District · PDF file11 L12text’ ... Solving’ Many’ problems ... predict’the’effects’of’forces’onmore’complex’systems’of’objects’and’to’analyze’objects

The value of displacement of A is approximately 4 m The value of displacement of B is approximately 3 m. ESSAY 24. ANS: a. The bicycle is moving with a constant speed, and motion is in the straight line, without any change in direction. The velocity of the bicycle is also constant. Therefore, the position-time graph will be a straight line. b. The average velocity of the bicycle is calculated as the slope of the position-time graph. c. The average speed of the bicycle is the absolute value of its average velocity. 25. ANS: a. The position-time graph of Linda’s motion from home to school is as follows:

b. Average velocity of Linda = Slope of the above graph =

c. During her return journey from school, the average velocity of Linda is –1.5 m/s.

A

B

Posi

tion

(m)

Time (s)1 2 3 4 5 6 7 8 9 10

1

2

3

4

5

6

7

8Po

sitio

n (m

)

Time (s)30 60 90 120 150 180 210 240

45

90

135

180

225

270

315

Page 46: Physics%–%1st%Quarter% - Lorain City School District · PDF file11 L12text’ ... Solving’ Many’ problems ... predict’the’effects’of’forces’onmore’complex’systems’of’objects’and’to’analyze’objects

26. ANS: a. The position-time table for the motion of the mouse from the kitchen to the store is:

Position vs Time Time (s) Position

(m) 0.0 0.0 0.5 2.0 1.0 4.0 1.5 6.0 2.0 8.0 2.5 10.0 3.0 12.0

b. The following particle model represents the motion of the mouse from the kitchen to the store. Kitchen Store

The time interval between two successive dots is of 0.5 s. c. The position-time graph corresponding to the position-time table formed in part a, is plotted below.

27. ANS: a. Thomas had been cycling for 1.0 s, when Anna started cycling. b. Anna crosses Thomas at a point 12.0 m from the starting point. c. Displacement of Thomas after 5.0 s = 20.0 m – 0.0 m = 20.0 m. Displacement of Anna after 5.0 s = 24.0 m – 0.0 m = 24.0 m. The difference in their positions after 5.0 s is 4.0 m with Anna being ahead. 28. ANS: a. The position of the train is 20 km in the east to station A. b. The position of the train becomes –10 km. c. The position of the train becomes –40 km.

Page 47: Physics%–%1st%Quarter% - Lorain City School District · PDF file11 L12text’ ... Solving’ Many’ problems ... predict’the’effects’of’forces’onmore’complex’systems’of’objects’and’to’analyze’objects

29. ANS: a.

b. There is the difference of sign in the two velocities. During the return journey, the displacement is negative. Therefore, the average velocity is also negative. c. The average velocity during the return journey also becomes positive. CHAPTER 2 ANSWER KEY WITH ANALYSIS MULTIPLE CHOICE 1. ANS: D Distance = speed × time Substituting the values in the relation for distance, we get, d = (15.0 m/s)(12.0 min)(60 s/1 min) = 10,800 m = 1.08 × 101 km DIF: 2 REF: p.p. 44 OBJ: 2.4.2 Differentiate between speed and velocity. TOP: Differentiate between speed and velocity. KEY: Average speed MSC: 2 NOT: /a/ Speed is multiplied with time to calculate the distance. All the quantities should have units of the same system. /b/ Time must be converted to seconds. /c/ Time should be multiplied with speed, not divided, to calculate the distance. /d/ Correct! 2. ANS: C Velocity and displacement are vector quantities. DIF: 1 REF: p.p. 35 OBJ: 2.2.3 Define displacement. TOP: Define displacement. KEY: Vectors MSC: 1 NOT: /a/ Speed and distance are scalar quantities. /b/ Velocity is a vector, but distance is scalar. /c/ Correct! /d/ Displacement is a vector, but speed is scalar.

Page 48: Physics%–%1st%Quarter% - Lorain City School District · PDF file11 L12text’ ... Solving’ Many’ problems ... predict’the’effects’of’forces’onmore’complex’systems’of’objects’and’to’analyze’objects

3. ANS: A When two displacement vectors have the same direction, their resultant has magnitude equal to the sum of their magnitudes. The direction of the resultant is the same as the direction of the original vectors. DIF: 2 REF: p.p. 37 OBJ: 2.2.3 Define displacement. TOP: Define displacement. KEY: Displacement, Resultant MSC: 2 NOT: /a/ Correct! /b/ The resultant is the sum of the vectors, and not the difference of the vectors. /c/ The resultant of two vectors, having the same direction, also has the same direction as the original vectors. /d/ The resultant is not perpendicular, but is in the direction of the original vectors. COMPLETION 4. ANS: particle DIF: 1 REF: p.p. 33 OBJ: 2.1.2 Develop a particle model to represent a moving object. STO: 9.D.D.3 TOP: Develop a particle model to represent a moving object. KEY: Particle model MSC: 2 5. ANS: resultant DIF: 1 REF: p.p. 35 OBJ: 2.2.5 Use a motion diagram to answer questions about an object's position or displacement. TOP: Use a motion diagram to answer questions about an object's position or displacement. KEY: Vectors, Resultant MSC: 1 SHORT ANSWER 6. ANS: Motion diagram is a series of images showing the positions of a moving object at equal time intervals. DIF: 1 REF: Pp.p. 33 OBJ: 2.1.1 Draw motion diagrams to describe motion. TOP: Draw motion diagrams to describe motion. KEY: Motion diagram MSC: 2 7. ANS:A coordinate system tells about the location of the zero point of the variables defining the motion of the object to be studied. The coordinate system also explains the direction in which the values of the variables increase. DIF: 1 REF: p.p. 34 OBJ: 2.2.1 Define coordinate systems for motion problems. TOP: Define coordinate systems for motion problems. KEY: Coordinate system, Origin

Page 49: Physics%–%1st%Quarter% - Lorain City School District · PDF file11 L12text’ ... Solving’ Many’ problems ... predict’the’effects’of’forces’onmore’complex’systems’of’objects’and’to’analyze’objects

MSC: 2 8. ANS:Displacement is a vector quantity, while distance is a scalar quantity. Displacement has both the magnitude and direction. Distance has only magnitude. The displacement of an object is equal to the difference between its final position and initial position. When an object starts moving from a certain point and after covering certain distance returns to its starting position, its displacement becomes zero. DIF: 2 REF: p.p. 37 OBJ: 2.2.3 Define displacement. TOP: Define displacement. KEY: Distance, Displacement, Scalars, Vectors MSC: 2 9. ANS:The equation of motion for average velocity is . In this equation, d is the position of an object, is the average velocity of the object, t is the time and di is the initial position of the object. According to this equation, an object’s position is equal to the average velocity multiplied by time plus the initial position. DIF: 1 REF: p.p. 47 OBJ: 2.4.3 Create pictorial, physical, and mathematical models of motion problems.; STO: 9.D.D.3 TOP: Create pictorial, physical, and mathematical models of motion problems.; KEY: Average velocity MSC: 2 10. ANS: Average velocity of an object is defined as the change in position of the object moving along a straight line, divided by the time during which the change occurred. In symbolic form, the average velocity

.

DIF: 2 REF: p.p. 44 OBJ: 2.4.1 Define velocity. TOP: Define velocity. KEY: Average velocity MSC: 2 11. ANS: The straight line position-time graph of a moving object gives the information about the nature of the motion of the object. It is also used to calculate the average velocity of the object. Since the graph is a straight line, the object is undergoing uniform motion. The slope of this line gives the value of the average velocity of the object. DIF: 2 REF: p.p. 43 OBJ: 2.4.1 Define velocity. TOP: Define velocity. KEY: Average velocity MSC: 12. ANS: The average velocity of an object is equal to the slope of the line representing the position-time graph of the object. In this graph, the slope of Ian’s line is steeper than the slope of Nick’s line. A steeper slope indicates a greater change in displacement during each time interval. Average velocity for a given time interval is proportional to the change in the displacement.

Page 50: Physics%–%1st%Quarter% - Lorain City School District · PDF file11 L12text’ ... Solving’ Many’ problems ... predict’the’effects’of’forces’onmore’complex’systems’of’objects’and’to’analyze’objects

Therefore, the average velocity of Ian is more than the average velocity of Nick. DIF: 3 REF: p.p. 43 OBJ: 2.4.1 Define velocity. TOP: Define velocity. KEY: Average velocity MSC: 2 13. ANS: The position-time graph corresponding to this particle model is plotted below.

DIF: 3 REF: p.p. 40 OBJ: 2.3.1 Develop position-time graphs for moving objects. TOP: Develop position-time graphs for moving objects. KEY: Position-time graph MSC: 2 14. ANS: In this case, motion is in the negative direction. Therefore, displacement and velocity of the bike are negative. Hence, both the displacement and the velocity have a negative sign. DIF: 1 REF: p.p. 44 OBJ: 2.4.1 Define velocity. TOP: Define velocity. KEY: Displacement, Average velocity MSC: 1 15. ANS: a. The value of displacement after 3 s is 12 m. b. After 5 s, the ball reaches at 20 m from its starting point. c. The position of the ball after 7 s = 28 m. The position of the ball after 9 s = 20 m. The direction of the motion of the ball is reversed after 7 s. Therefore, the displacement of the ball between 7 s and 9 s = Δd = 20 – 28 = –8 m. The negative sign shows that the direction of displacement in this case is negative, toward the starting point. DIF: 3 REF: p.p. 36 OBJ: 2.2.5 Use a motion diagram to answer questions about an object's position or displacement. TOP: Use a motion diagram to answer questions about an object's position or displacement.

Time (s)

Posi

tion

(m)

2 4 6 8 10 12 14 16 18 20 22 24

5

10

15

20

25

30

35

40

45

50

Page 51: Physics%–%1st%Quarter% - Lorain City School District · PDF file11 L12text’ ... Solving’ Many’ problems ... predict’the’effects’of’forces’onmore’complex’systems’of’objects’and’to’analyze’objects

KEY: Displacement MSC: 3 16. ANS: The point of intersection of the two graphs is 150 m at about 38 s. Swimmer B passes Swimmer A about 150 m beyond the origin 38 s after A has passed the origin. DIF: 2 REF: p.p. 38 OBJ: 2.2.4 Determine a time interval. TOP: Determine a time interval. KEY: Time interval, Origin MSC: 2 17. ANS: The position-time graph of the pedestrian intersects the x-axis at 2.5 s. Thus, the displacement in 2.5 s is –10 m. DIF: 2 REF: p.p. 38 OBJ: 2.3.2 Use a position-time graph to interpret an object's position or displacement. TOP: Use a position-time graph to interpret an object's position or displacement. KEY: Position-time graph, Displacement MSC: 1 18. ANS: The points on the line of a position-time graph of an object represent the most likely positions of the moving object at the times between the recorded data points. DIF: 2 REF: p.p. 38 OBJ: 2.3.2 Use a position-time graph to interpret an object's position or displacement. TOP: Use a position-time graph to interpret an object's position or displacement. KEY: Position-time graph, Position MSC: 1 19. ANS:

The resultant displacement is 2 units toward the east, and his position is at 2. DIF: 2 REF: p.p. 34 OBJ: 2.2.5 Use a motion diagram to answer questions about an object's position or displacement. TOP: Use a motion diagram to answer questions about an object's position or displacement. KEY: Displacement MSC: 2

Page 52: Physics%–%1st%Quarter% - Lorain City School District · PDF file11 L12text’ ... Solving’ Many’ problems ... predict’the’effects’of’forces’onmore’complex’systems’of’objects’and’to’analyze’objects

20. ANS:The two attributes of the coordinate system chosen for a motion diagram are the origin and the axis of the coordinate system. DIF: 2 REF: p.p. 34 OBJ: 2.2.5 Use a motion diagram to answer questions about an object's position or displacement. TOP: Use a motion diagram to answer questions about an object's position or displacement. KEY: Coordinate system, Origin MSC: 2 21. ANS:The time interval is a scalar quantity. Its value is not affected by the change in the relative position of the origin of the coordinate system. DIF: 2 REF: p.p. 36 OBJ: 2.2.4 Determine a time interval. TOP: Determine a time interval. KEY: Time interval MSC: 2 22. ANS:Speed is given in km/h, therefore convert time in minutes to hours.

distance traveled = (speed)(time taken) = (35 km/h)(12 min)(1 h/60 min) = 7.0 km DIF: 2 REF: p.p. 36 OBJ: 2.2.3 Define displacement. TOP: Define displacement. KEY: Distance MSC: 2 23. ANS:The displacements of A and B are the coordinates of the vertical line with their respective position-time graphs at 4 s.

The value of displacement of A is approximately 4 m The value of displacement of B is approximately 3 m. DIF: 3 REF: p.p. 36 OBJ: 2.2.3 Define displacement. TOP: Define displacement. KEY: Displacement MSC: 1

A

B

Posi

tion

(m)

Time (s)1 2 3 4 5 6 7 8 9 10

1

2

3

4

5

6

7

8

Page 53: Physics%–%1st%Quarter% - Lorain City School District · PDF file11 L12text’ ... Solving’ Many’ problems ... predict’the’effects’of’forces’onmore’complex’systems’of’objects’and’to’analyze’objects

ESSAY: 24. ANS: a. The bicycle is moving with a constant speed, and motion is in the straight line, without any change in direction. The velocity of the bicycle is also constant. Therefore, the position-time graph will be a straight line. b. The average velocity of the bicycle is calculated as the slope of the position-time graph. c. The average speed of the bicycle is the absolute value of its average velocity. DIF: 3 REF: p.p. 44 OBJ: 2.4.2 Differentiate between speed and velocity. TOP: Differentiate between speed and velocity. KEY: Average speed, Average velocity MSC: 2 25. ANS: a. The position-time graph of Linda’s motion from home to school is as follows:

b. Average velocity of Linda = Slope of the above graph =

c. During her return journey from school, the average velocity of Linda is –1.5 m/s. DIF: 3 REF: p.p. 43 OBJ: 2.4.1 Define velocity. TOP: Define velocity. KEY: Average velocity MSC: 3 26. ANS: a. The position-time table for the motion of the mouse from the kitchen to the store is:

Position vs Time Time (s) Position

(m) 0.0 0.0 0.5 2.0 1.0 4.0

Posi

tion

(m)

Time (s)30 60 90 120 150 180 210 240

45

90

135

180

225

270

315

Page 54: Physics%–%1st%Quarter% - Lorain City School District · PDF file11 L12text’ ... Solving’ Many’ problems ... predict’the’effects’of’forces’onmore’complex’systems’of’objects’and’to’analyze’objects

1.5 6.0 2.0 8.0 2.5 10.0 3.0 12.0

b. The following particle model represents the motion of the mouse from the kitchen to the store. Kitchen Store

The time interval between two successive dots is of 0.5 s. c. The position-time graph corresponding to the position-time table formed in part a, is plotted below.

DIF: 3 REF: p.p. 40 OBJ: 2.3.3 Make motion diagrams, pictorial representations, and position-time graphs that are equivalent representations describing an object's motion.; TOP: Make motion diagrams, pictorial representations, and position-time graphs that are equivalent representations describing an object's motion.; KEY: Position-time graph MSC: 2 27. ANS: a. Thomas had been cycling for 1.0 s, when Anna started cycling. b. Anna crosses Thomas at a point 12.0 m from the starting point. c. Displacement of Thomas after 5.0 s = 20.0 m – 0.0 m = 20.0 m. Displacement of Anna after 5.0 s = 24.0 m – 0.0 m = 24.0 m. The difference in their positions after 5.0 s is 4.0 m with Anna being ahead. DIF: 3 REF: p.p. 42 OBJ: 2.3.2 Use a position-time graph to interpret an object's position or displacement. TOP: Use a position-time graph to interpret an object's position or displacement. KEY: Position-time graph, Instantaneous position MSC: 2

Page 55: Physics%–%1st%Quarter% - Lorain City School District · PDF file11 L12text’ ... Solving’ Many’ problems ... predict’the’effects’of’forces’onmore’complex’systems’of’objects’and’to’analyze’objects

28. ANS: a. The position of the train is 20 km in the east to station A. b. The position of the train becomes –10 km. c. The position of the train becomes –40 km. DIF: 3 REF: p.p. 35 OBJ: 2.2.2 Recognize that the chosen coordinate system affects the sign of objects' positions. TOP: Recognize that the chosen coordinate system affects the sign of objects' positions. KEY: Coordinate system, Position, Origin MSC: 3 29. ANS: a.

b. There is the difference of sign in the two velocities. During the return journey, the displacement is negative. Therefore, the average velocity is also negative. c. The average velocity during the return journey also becomes positive. DIF: 3 REF: p.p. 38 OBJ: 2.3.1 Develop position-time graphs for moving objects. TOP: Develop position-time graphs for moving objects. KEY: Displacement, Position-time graph, Average velocity MSC: 2

Page 56: Physics%–%1st%Quarter% - Lorain City School District · PDF file11 L12text’ ... Solving’ Many’ problems ... predict’the’effects’of’forces’onmore’complex’systems’of’objects’and’to’analyze’objects

Name ___________________________________________________ Period _______________

Chapter 3 Test MULTIPLE CHOICE Identify the letter of the choice that best completes the statement or answers the question. ____1. A man starts his car from rest and accelerates at 1 m/s2 for 2 seconds. He then continues at a constant velocity for 10 seconds until he sees a tree blocking the road and applies brakes. The car, decelerating at 1 m/s2, finally comes to rest. Which of the following graphs represents the motion correctly? a.

c.

b.

d.

v

t5 10 15 20 25 30 35 40 (s)

2468

1012141618202224262830

(m/s) v

t5 10 15 20 25 30 35 (s)

2468

101214161820222426283032343638

(m/s)

v

t5 10 15 20 25 (s)

2

4

6

8

10 (m/s) v

t5 10 15 20 25 (s)

2

4

6

8

10 (m/s)

Page 57: Physics%–%1st%Quarter% - Lorain City School District · PDF file11 L12text’ ... Solving’ Many’ problems ... predict’the’effects’of’forces’onmore’complex’systems’of’objects’and’to’analyze’objects

SHORT ANSWER 2. The velocity-time graph of a car’s motion is given below. Plot the corresponding acceleration-time graph.

PROBLEM 3. The velocity-time graph of the motion of a particle is shown below. Calculate the total displacement of the particle from 0 to 29 seconds.

4. A car starts from rest with an acceleration of 2.84 m/s2 at the instant when a second car moving with a velocity of 25.7 m/s passes it in a parallel line. How far does the first car move before it overtakes the second car? 5. A ball is thrown vertically upward with a speed of 1.53 m/s from a point 4.21 m above the ground. Calculate the time in which the ball will reach the ground.

v

t5 10 15 (s)

5

10

–5

–10

(m/s)

v (m/s)

t (s)2 4 6 8 10 12 14 16 18 20 22 24 26 28 30 32 34 36

2468

10121416182022242628303234

Page 58: Physics%–%1st%Quarter% - Lorain City School District · PDF file11 L12text’ ... Solving’ Many’ problems ... predict’the’effects’of’forces’onmore’complex’systems’of’objects’and’to’analyze’objects

6. A ball is thrown vertically upward with a speed of 1.86 m/s from a point 3.82 m above the ground. Calculate the time in which the ball will reach the ground. 7. Julia throws a ball vertically upward from the ground with a speed of 5.89 m/s. Andrew catches it when it is on its way down at a height of 1.27 m from the ground. After how much time does Andrew catch the ball? 8. The graph below shows how the velocity of a rolling ball changes with time. Calculate the acceleration of the ball.

9. A car accelerates from rest at 5 m/s2 for 5 seconds. It moves with a constant velocity for some time, and then decelerates at 5 m/s2 to come to rest. The entire journey takes 25 seconds. Plot the velocity-time graph of the motion.

(m/s)v

t2 4 6 8 10 12 14 16 18 (s)

1

2

3

4

5

6

7

8

9

10

Page 59: Physics%–%1st%Quarter% - Lorain City School District · PDF file11 L12text’ ... Solving’ Many’ problems ... predict’the’effects’of’forces’onmore’complex’systems’of’objects’and’to’analyze’objects

10. The graph below represents the velocity-time variation of a car’s motion.

Use the graph to find: a) The acceleration of the car between t = 0 s and t = 5 s.

b) The acceleration of the car between t = 5 s and t = 10 s.

c) The acceleration of the car between t = 10 s and t = 20 s.

d) The acceleration of the car between t = 20 s and t = 30 s.

11. A boy throws a ball vertically upward with a speed of 19 m/s. Calculate the speed of the ball

when it is at a height equal to 0.77 times the maximum height reached by the ball.

v

t5 10 15 20 25 30 (s)

5

10

15

(m/s)

Page 60: Physics%–%1st%Quarter% - Lorain City School District · PDF file11 L12text’ ... Solving’ Many’ problems ... predict’the’effects’of’forces’onmore’complex’systems’of’objects’and’to’analyze’objects

Chapter 3 ANSWER KEY MULTIPLE CHOICE 1.ANS: B SHORT ANSWER 2. ANS:

PROBLEM 3. ANS: 532.5 m 4. ANS: 465 m 5. ANS: 1.10 s 6. ANS: 1.093 s 7. ANS: 0.92 s 8. ANS: 4 m/s2 9. ANS:

10. ANS: a) 0.80 m/s2 b) 0 m/s2 c) 1 m/s2 d) –1.6 m/s2 11. ANS: 9.1 m/s

a

t1 2 3 4 5 6 7 8 9 (s)

1

2

3

4

5

6

–1

–2

–3

–4

–5

–6

(m/s^2)

v

t5 10 15 20 25 30 (s)

5

10

15

20

25

30

(m/s)

Page 61: Physics%–%1st%Quarter% - Lorain City School District · PDF file11 L12text’ ... Solving’ Many’ problems ... predict’the’effects’of’forces’onmore’complex’systems’of’objects’and’to’analyze’objects

ANSWER KEY ANALYSIS MULTIPLE CHOICE 1. ANS: B DIF: 3 REF: p.p. 57 OBJ: 3.1.3 Create velocity-time graphs. TOP: Create velocity-time graphs. KEY: Velocity-time graph MSC: 3 NOT: /A/ This graph does not show a period of constant velocity. /B/ Correct! /C/ The car starts from rest instead of at constant speed and the slopes of the different parts of the graph are incorrect./D/ The car moves at a constant velocity for 12 seconds. SHORT ANSWER 2. ANS:

DIF: 3 REF: p.p. 66 OBJ: 3.1.2 Relate velocity and acceleration to the motion of an object. TOP: Relate velocity and acceleration to the motion of an object. KEY: Relate velocity and acceleration MSC: 3 PROBLEM 3. ANS: 532.5 m DIF: 3 REF: p.p. 65 OBJ: 3.2.1 Interpret position-time graphs for motion with constant acceleration. TOP: Interpret position-time graphs for motion with constant acceleration. KEY: Position-time graph MSC: 3 NOT: The area under the graph gives the distance traveled. 4. ANS: 465 m DIF: 3 REF: p.p. 65

a

t1 2 3 4 5 6 7 8 9 (s)

1

2

3

4

5

6

–1

–2

–3

–4

–5

–6

(m/s^2)

Page 62: Physics%–%1st%Quarter% - Lorain City School District · PDF file11 L12text’ ... Solving’ Many’ problems ... predict’the’effects’of’forces’onmore’complex’systems’of’objects’and’to’analyze’objects

OBJ: 3.2.2 Determine mathematical relationships among position, velocity, acceleration, and time.; TOP: Determine mathematical relationships among position, velocity, acceleration, and time.; KEY: Position, Velocity, Acceleration, Time MSC: 3 NOT: The distance covered by both cars is the same when the second car just overtakes the first car. 5. ANS: 1.10 s DIF: 3 REF: p.p. 72 OBJ: 3.3.2 Solve objects involving objects in free fall. TOP: Solve objects involving objects in free fall. KEY: Free fall MSC: 3 NOT: The total time is the sum of the time taken by the ball to reach its topmost point and the time to come down from the topmost point to the ground. 6. ANS: 1.093 s DIF: 3 REF: p.p. 72 OBJ: 3.3.2 Solve objects involving objects in free fall. TOP: Solve objects involving objects in free fall. KEY: Free fall MSC: 3 NOT: The total time is the sum of the time taken by the ball to reach its topmost point and the time to come down from the topmost point to the ground. 7. ANS: 0.92 s DIF: 3 REF: p.p. 72 OBJ: 3.3.2 Solve objects involving objects in free fall. TOP: Solve objects involving objects in free fall. KEY: Free fall MSC: 3 NOT: The total time is the sum of the time taken by the ball to reach its topmost point and the time to come down from the topmost point to the point where the ball is caught. 8. ANS: 4 m/s2 DIF: 3 REF: p.p. 59 OBJ: 3.1.1 Define acceleration. STO: 9.C.C.23 TOP: Define acceleration. KEY: Acceleration MSC: 3 NOT: Acceleration is the rate of change of velocity. 9. ANS:

DIF: 3 REF: p.p. 59 OBJ: 3.1.3 Create velocity-time graphs.

v

t5 10 15 20 25 30 (s)

5

10

15

20

25

30

(m/s)

Page 63: Physics%–%1st%Quarter% - Lorain City School District · PDF file11 L12text’ ... Solving’ Many’ problems ... predict’the’effects’of’forces’onmore’complex’systems’of’objects’and’to’analyze’objects

TOP: Create velocity-time graphs. KEY: Velocity-time graph MSC: 3 NOT: The velocity-time graph is a straight line making an acute angle with the positive x-axis for accelerated motion, a horizontal line for uniform motion, and a straight line making an obtuse angle with the positive x-axis for decelerated motion. 10. ANS: a) 0.80 m/s2 b) 0 m/s2 c) 1 m/s2 d) –1.6 m/s2 DIF: 3 REF: p.p. 66 OBJ: 3.2.3 Apply graphical and mathematical relationships to solve problems related to constant acceleration. TOP: Apply graphical and mathematical relationships to solve problems related to constant acceleration. KEY: Problems related to constant acceleration MSC: 3 NOT: The acceleration can be obtained by calculating the slope of the velocity-time graph. 11. ANS: 9.1 m/s DIF: 3 REF: p.p. 72 OBJ: 3.3.1 Define acceleration due to gravity. STO: 9.C.B.13 TOP: Define acceleration due to gravity. KEY: Acceleration due to gravity MSC: 3 NOT: Calculate the maximum height reached and then use the starting point as the highest point.

*END  OF  Exams  1-­‐>  3*  

 

Page 64: Physics%–%1st%Quarter% - Lorain City School District · PDF file11 L12text’ ... Solving’ Many’ problems ... predict’the’effects’of’forces’onmore’complex’systems’of’objects’and’to’analyze’objects

Chapter 4 Test Multiple Choices Identify the letter of the choice that best completes the statement or answers the question. ____1. Which of the following system of forces provides the block the highest net force?

a.

11 N 71 N

c.

227 N 153 N

b.

405 N 403 N

d.

22.7 N 15.3 N PROBLEM 2. A car of mass 1330 kg is traveling at 28.0 m/s. The driver applies the brakes to bring the car

to rest over a distance of 79.0 m. Calculate the retarding force acting on the car. 3. Two men pull a 31-kg box with forces 9.7 N and 7.6 N in the directions shown below. Find

the resultant acceleration of the box and the direction in which the box moves.

9.7 N 7.6 N 4. An elevator is moving down with an acceleration of 3.36 m/s2. What would be the apparent

weight of a 64.2-kg man in the elevator? 5. An elevator is moving down with an acceleration of 1.40 m/s2. A 14.5-kg block hangs from a Spring balance fixed to the roof of the elevator. What is the apparent weight of the block? 6. The blocks shown below are placed on a smooth horizontal surface and connected by a piece

of string. If a 8.8-N force is applied to the 8.8-kg block, what is the tension in the string? 8.8 N

19.4 kg 8.8 kg

Page 65: Physics%–%1st%Quarter% - Lorain City School District · PDF file11 L12text’ ... Solving’ Many’ problems ... predict’the’effects’of’forces’onmore’complex’systems’of’objects’and’to’analyze’objects

7. Three blocks A, B, and C are connected by two massless strings passing over smooth pulleys as shown below, with the 3.4-kg block on a smooth horizontal surface. Calculate the tension in the strings connecting A and B, and, B and C.

3.4 kg

6.9 kg 4.1 kg 8. A 2.1-kg block is kept on a 3.5-kg block resting on the floor of an elevator. If the elevator is

moving up at 1.1 m/s2, calculate the following: a. Force exerted by the 2.1-kg block on the 3.5-kg block. b. Force exerted by the floor of the elevator on the 3.5-kg block. 9. Two blocks A and B are kept in contact with each other on a smooth horizontal surface and a

3.9-N force is applied to the block A as shown below. Calculate the force exerted by the block A on the block B, assuming that they do not separate at any time. The mass of block A is 4.4 kg and that of block B is 5.5 kg.

3.9 N

10. Raindrops fall on Brian’s head at the rate of 4 drops per second. Each raindrop has a mass of

1.6 mg and falls with a speed of 25 m/s. assuming that on making contact with Brian’s head the drops come to rest and do not rebound, calculate the force felt by Brian.

Page 66: Physics%–%1st%Quarter% - Lorain City School District · PDF file11 L12text’ ... Solving’ Many’ problems ... predict’the’effects’of’forces’onmore’complex’systems’of’objects’and’to’analyze’objects

Chapter4 ANSWER KEY

MULTIPLE CHOICES 1. ANS: A PROBLEM

2. ANS: N

3. ANS: m/s2 to left

4. ANS: 413 N

5. ANS: 12.4 N

6. ANS: 6.1 N

7. ANS: Tension in the string connecting A and B = 54 N Tension in the string connecting B and C = 48 N

8. ANS:

a. Force exerted by the 2.1-kg block on the 3.5-kg block = 23 N b. Force exerted by the floor of the elevator on the 3.5-kg block = 61 N

9. ANS 2.2 N 10. ANS: N

ANSWER KEY ANALYSIS MULTIPLE CHOICES

1. ANS: A DIF: 1 REF: p.p. 87 OBJ: 4.1.1 Define force. TOP: Define force. KEY: Force MSC: 1 NOT: /A/ Correct! /B/ the difference between the forces is not the maximum./C/ the net force is the difference between the two forces. /D/ this system does not give the highest net force. PROBLEM

2. ANS: N DIF: 3 REF: p.p. 87 OBJ: 4.1.2 Apply Newton's second law to solve

6.60× 103

6.8× 10−2

1.6× 10−4

6.60× 103

Page 67: Physics%–%1st%Quarter% - Lorain City School District · PDF file11 L12text’ ... Solving’ Many’ problems ... predict’the’effects’of’forces’onmore’complex’systems’of’objects’and’to’analyze’objects

problems. STO: C.11-12.D, 12.C.2.5 TOP: Apply Newton's second law to solve problems. KEY: Newton's second law MSC: 3 NOT: Calculate the acceleration using the initial and final velocities. The product of the mass and acceleration gives the force.

3. ANS: m/s2 to left DIF: 3 REF: p.p. 87 OBJ: 4.1.2 Apply Newton's second law to solve problems. STO: C.11-12.D, 12.C.2.5 TOP: Apply Newton's second law to solve problems. KEY: Newton's second law MSC: 3 NOT: The resultant acceleration is obtained on dividing the net force by the mass of the box.

4. ANS: 413 N DIF: 3 REF: p.p. 97 OBJ: 4.2.2 Differentiate between actual weight and apparent weight. TOP: Differentiate between actual weight and apparent weight. KEY: Apparent weight MSC: 3 NOT: Use Newton's second law to find the apparent weight of the man.

5. ANS: 12.4 N DIF: 2 REF: p.p. 96 OBJ: 4.2.2 Differentiate between actual weight and apparent weight. TOP: Differentiate between actual weight and apparent weight. KEY: Apparent weight MSC: 3 NOT: Use Newton's second law to find the apparent weight of the block.

6. ANS: 6.1 N DIF: 3 REF: p.p. 102 OBJ: 4.3.2 Explain the tension in ropes and strings in terms of Newton's third law. TOP: Explain the tension in ropes and strings in terms of Newton's third law. KEY: Tension MSC: 3 NOT: Draw the free-body diagram for each block and then use Newton's second law to find the tension.

7. ANS: Tension in the string connecting A and B = 54 N Tension in the string connecting B and C = 48 N DIF: 3 REF: p.p. 102 OBJ: 4.3.2 Explain the tension in ropes and strings in terms of Newton's third law. TOP: Explain the tension in ropes and strings in terms of Newton's third law. KEY: Tension MSC: 3 NOT: Draw the free-body diagram for each block and then use Newton's second law to find the

6.8× 10−2

Page 68: Physics%–%1st%Quarter% - Lorain City School District · PDF file11 L12text’ ... Solving’ Many’ problems ... predict’the’effects’of’forces’onmore’complex’systems’of’objects’and’to’analyze’objects

tensions.

8. ANS: a. Force exerted by the 2.1-kg block on the 3.5-kg block = 23 N b. Force exerted by the floor of the elevator on the 3.5-kg block = 61 N DIF: 3 REF: p.p. 102 OBJ: 4.3.4 Determine the value of the normal force by applying Newton's second law. STO: C.11-12.D, 12.C.2.5 TOP: Determine the value of the normal force by applying Newton's second law. KEY: Normal force MSC: 3 NOT: Draw the free-body diagram for each block and then use Newton's second law to find the interaction forces.

9. ANS 2.2 N DIF: 3 REF: p.p. 102 OBJ: 4.3.4 Determine the value of the normal force by applying Newton's second law. STO: C.11-12.D, 12.C.2.5 TOP: Determine the value of the normal force by applying Newton's second law. KEY: Normal force MSC: 3 NOT: Draw the free-body diagram for each block and then use Newton's second law to find the interaction force.

10. ANS: N DIF: 2 REF: P.P. 87 OBJ: 4.1.1 Define force. TOP: Define force. KEY: Force MSC: 3 NOT: The rate of change of momentum is equal to the force.

1.6× 10−4

Page 69: Physics%–%1st%Quarter% - Lorain City School District · PDF file11 L12text’ ... Solving’ Many’ problems ... predict’the’effects’of’forces’onmore’complex’systems’of’objects’and’to’analyze’objects

Name ___________________________________________________ Period _________________

Chapter 5 Test PROBLEM 1. The magnitude of the force is 57 N, of force is 57 N, and of is 57 N. The angles

and are 30° each. Use the graphical method to find the resultant of the forces , and .

2. A vector has magnitude 58.0 units and is inclined to the positive axis of x at 42.0°. Vector

has magnitude 36.0 units and is inclined to the positive axis of x at 121°. Vector has magnitude 40.0 units and is inclined to the positive axis of x at 53.0°. Use the graphical method to find the resultant of the vectors , and .

3. A river flows at a speed of 4.60 m/s. A boat, capable of moving with a speed of 5.80 m/s in

still water is rowed across the river at an angle of 53.0° to the river flow. Calculate the resultant velocity with which the boat moves and the angle that its resultant motion makes to the river flow.

4. What should be the angle between two vectors of magnitudes 3.20 and 5.70 units, so that their

resultant has a magnitude of 6.10 units? 5. An airplane has to fly eastward to a destination 856 km away. If wind is blowing at 18.0 m/s

northward and the air speed of the plane is 161 m/s, in what direction should the plane head to reach its destination?

6. A 2.7-kg box is released on a horizontal surface with an initial speed of 2.9 m/s. It moves on

the surface with a deceleration of 0.27 m/s2. Calculate the kinetic friction force on the box.

Page 70: Physics%–%1st%Quarter% - Lorain City School District · PDF file11 L12text’ ... Solving’ Many’ problems ... predict’the’effects’of’forces’onmore’complex’systems’of’objects’and’to’analyze’objects

7. A block of 7.80 kg kept on an inclined plane just begins to slide at an angle of inclination of 35.0°. Once it has been set into motion, the angle is reduced to 30.0° to keep the block moving at constant speed. Calculate the maximum force of static friction and the force of kinetic friction for the surfaces in contact.

8. The system shown below is in equilibrium. Calculate the force of friction acting on the block

A. The mass of block A is 4.10 kg and that of block B is 4.30 kg. The angle q is 33.0°.

9. The blocks A and B are connected by a piece of spring. Block B rests on an inclined plane of

40° and blocks a hangs vertically. The coefficient of friction between block A and the inclined plane is 0.29. Calculate the acceleration of the system if the mass of block A is 0.15 kg and that of block B is 7.5 kg.

10. The blocks A and B are connected by a piece of spring. Block B rests on a smooth inclined

plane of 35° and blocks a hangs vertically. Calculate the acceleration of the system if the mass of block A is 1.06 kg and that of block B is 6.4 kg.

Page 71: Physics%–%1st%Quarter% - Lorain City School District · PDF file11 L12text’ ... Solving’ Many’ problems ... predict’the’effects’of’forces’onmore’complex’systems’of’objects’and’to’analyze’objects

ANSWER KEY PROBLEM

1. ANS: Zero 0

2. ANS: The resultant is 113 units at an angle of 64.4° to the positive axis of x.

3. ANS: The boat moves with 9.32 m/s at 29.8° to the river flow.

4. ANS: 98.7°

5. ANS: 6.42° south of east 6. ANS: 0.73 N

7. ANS: The maximum force of static friction is 43.8 N and the force of kinetic friction is 38.2 N.

8. ANS: 64.9 N

9. ANS: 3.8 m/s2

10. ANS: 3.4 m/s2

Page 72: Physics%–%1st%Quarter% - Lorain City School District · PDF file11 L12text’ ... Solving’ Many’ problems ... predict’the’effects’of’forces’onmore’complex’systems’of’objects’and’to’analyze’objects

ANSWER KEY ANALYSIS PROBLEM 1. ANS: Zero 0 DIF: 3 REF: p.p. 120 OBJ: 5.1.1 Evaluate the sum of two or more vectors in two dimensions graphically. TOP: Evaluate the sum of two or more vectors in two dimensions graphically. KEY: Sum of vectors MSC: 3 NOT: Draw the vectors to scale and use the protractor to draw them tip to tail. The closing side of the polygon in reverse order is the resultant.

2. ANS: The resultant is 113 units at an angle of 64.4° to the positive axis of x. DIF: 3 REF: p.p. 120 OBJ: 5.1.1 Evaluate the sum of two or more vectors in two dimensions graphically. TOP: Evaluate the sum of two or more vectors in two dimensions graphically. KEY: Sum of vectors MSC: 3 NOT: Draw the vectors to scale and use the protractor to draw them tip to tail. The closing side of the polygon in reverse order is the resultant. Measure the length of the resultant to get the magnitude of the vector, and use a protractor to find the direction of the resultant.

3. ANS: The boat moves with 9.32 m/s at 29.8° to the river flow. DIF: 3 REF: p.p. 122 OBJ: 5.1.3 Solve for the sum of two or more vectors by adding the components of the vectors. TOP: Solve for the sum of two or more vectors by adding the components of the vectors. KEY: Sum of vectors MSC: 3 NOT: Resolve the velocity of the boat into components along and perpendicular to the river flow.

4. ANS: 98.7° DIF: 3 REF: p.p. 122 OBJ: 5.1.3 Solve for the sum of two or more vectors by adding the components of the vectors. TOP: Solve for the sum of two or more vectors by adding the components of the vectors. KEY: Sum of vectors MSC: 3 NOT: Resolve one of the vectors into components along and perpendicular to the second vector.

5. ANS: 6.42° south of east DIF: 3 REF: p.p. 122 OBJ: 5.1.3 Solve for the sum of two or more vectors by adding the components of the vectors.

Page 73: Physics%–%1st%Quarter% - Lorain City School District · PDF file11 L12text’ ... Solving’ Many’ problems ... predict’the’effects’of’forces’onmore’complex’systems’of’objects’and’to’analyze’objects

TOP: Solve for the sum of two or more vectors by adding the components of the vectors. KEY: Sum of vectors MSC: 3 NOT: Resolve the velocity of the plane in and perpendicular to the direction of the wind. 6. ANS: 0.73 N DIF: 2 REF: p.p. 127 OBJ: 5.2.1 Define the friction force. TOP: Define the friction force. KEY: Friction MSC: 2 NOT: The only force on the box is the kinetic friction, which should be equal to the product of mass and acceleration of the box.

7. ANS: The maximum force of static friction is 43.8 N and the force of kinetic friction is 38.2 N. DIF: 3 REF: p.p. 127 OBJ: 5.2.2 Distinguish between static and kinetic friction. TOP: Distinguish between static and kinetic friction. KEY: Inclined plane MSC: 3 NOT: The maximum force of static friction is equal to the force that just moves a static object. The kinetic friction force is equal to the force that keeps it moving at constant speed.

8. ANS: 64.9 N DIF: 2 REF: p.p. 131 OBJ: 5.3.1 Determine the force that produces equilibrium when three forces act on an object. TOP: Determine the force that produces equilibrium when three forces act on an object. KEY: Equilibrant MSC: 3 NOT: The tension in the three strings keeps the knot in equilibrium. The tension in the horizontal string is equal to the friction on the block A.

9. ANS: 3.8 m/s2 DIF: 3 REF: p.p. 132 OBJ: 5.3.2 Analyze the motion of an object on an inclined plane with and without friction. TOP: Analyze the motion of an object on an inclined plane with and without friction. KEY: Inclined plane MSC: 3 NOT: Resolve all forces parallel and perpendicular to the inclined plane. The friction force opposes the motion of the block B.

10. ANS: 3.4 m/s2 DIF: 3 REF: p.p. 132 OBJ: 5.3.2 Analyze the motion of an object on an inclined plane with and without friction. TOP: Analyze the motion of an object on an inclined plane with and without friction. KEY: Inclined plane MSC: 3 NOT: Resolve all forces parallel and perpendicular to the inclined plane.

Page 74: Physics%–%1st%Quarter% - Lorain City School District · PDF file11 L12text’ ... Solving’ Many’ problems ... predict’the’effects’of’forces’onmore’complex’systems’of’objects’and’to’analyze’objects

Name ____________________________________________Period _________________

Chapter 6 Test MULTIPLE CHOICES Identify the letter of the choice that best completes the statement or answers the question.

____1. The path of a projectile through space is called its: a. equilibrant c. range b. torque d. trajectory ____2. A soldier throws a grenade horizontally from the top of a cliff. Which of the following

curves best describes the path taken by the grenade? a. Circle c. Hyperbola b. Ellipse d. Parabola ____3. A stone is thrown horizontally from the top of a 25.00-m cliff. The stone lands at a

distance of 40.00 m from the edge of the cliff. What is the initial horizontal velocity of the stone?

a. 2.260 m/s c. 17.70 m/s b. 15.60 m/s d. 22.05 m/s ____4. A ball is thrown horizontally at 10.0 m/s from the top of a hill 50.0 m high. How far from

the base of the cliff, would the ball hit the ground? a. 23.6 m c. 31.9 m b. 26.4 m d. 45.0 m ____5. A ball is thrown horizontally from a hill 29.0 m high at a velocity of 4.00 m/s. Find the

distance between the base of the hill and the point where the ball hits the ground. a. 2.43 m c. 10.06 m b. 9.73 m d. 3.28 m ____6. A player kicks a football at an angle of 30.0° above the horizontal. The football has an

initial velocity of 20.0 m/s. Find the horizontal component of the velocity and the maximum height attained by the football.

a. 10.0 m/s, 17.6 m c. 25.1 m/s, 7.40 m b. 17.3 m/s, 5.10 m d. 30.3 m/s, 9.50 m

Page 75: Physics%–%1st%Quarter% - Lorain City School District · PDF file11 L12text’ ... Solving’ Many’ problems ... predict’the’effects’of’forces’onmore’complex’systems’of’objects’and’to’analyze’objects

____7. A missile launches at a velocity of 30.0 m/s at an angle of 30.0° to the normal. What is

the maximum height the missile attains? a. 11.5 m c. 34.4 m b. 27.5 m d. 45.9 m ____8. A player kicks a football with an initial velocity of 3.00 m/s at an angle of 60.0° above the horizontal. What is the horizontal distance traveled by the football? a. 0.312 m c. 0.673 m b. 0.397 m d. 0.795 m ____9. The movement of an object or a point mass at a constant speed around a circle that has a

fixed radius is called uniform: a. circular motion c. elliptical motion b. parabolic motion d. rotational motion ____10. A sprinter runs at a speed of 3.00 m/s on a circular track that has a radius of 40.00 m.

Find the centripetal acceleration of the sprinter. a. 0.225 m/s2 c. 0.750 m/s2 b. 4.44 m/s2 d. 0.0750 m/s2 ____11. A 0.50-kg ball is attached to a string of 0.50 m and swung in a horizontal circle with a

velocity of 1.0 m/s. Find the centripetal force of the ball. a. 0.50 N c. 2.0 N b. 1.0 N d. 2.5 N ____12. A 1.00-kg ball is attached to a string of 0.50 m and swung in a horizontal circle with a

velocity of 2.00 m/s. Find the centripetal acceleration. a. 0.25 m/s2 c. 4.0 m/s2 b. 2.0 m/s2 d. 8.0 m/s2 SHORT ANSWERS 13. An object in uniform circular motion moves at a constant speed around a circle with a fixed

radius. Why is the object said to be accelerating though it has a constant speed? PROBLEMS: 14. A hiker throws a ball at an angle of 21.0° above the horizontal from a hill 21.0 m high. The

hiker’s height is 1.750 m. The magnitudes of the horizontal and vertical components of the velocity are 14.004 m/s and 5.376 m/s, respectively. Find the distance between the base of the hill and the point where the ball hits the ground. (Consider the hiker’s height while calculating the answer.)

Page 76: Physics%–%1st%Quarter% - Lorain City School District · PDF file11 L12text’ ... Solving’ Many’ problems ... predict’the’effects’of’forces’onmore’complex’systems’of’objects’and’to’analyze’objects

15. A ball is tied to an elastic string of length 8.0 m and swung in a horizontal circle with a velocity of 0.8 m/s. When a metallic object is tied to a rope of length 2.75 m and swung in a horizontal circle, it makes one revolution in 2.9 s. The ratio of the centripetal force in the

string to the centripetal force in the rope is . Find the mass of the metallic object attached

to the rope, if the centripetal force in the string is 0.20 N.

16. Two grenades, A and B, are thrown horizontally with different speeds from the top of a cliff 70 m high. The speed of A is 2.50 m/s and the speed of B is 3.40 m/s. Both grenades remain

in air for 3.77 s. Assume that the acceleration due to gravity is 9.86 m/s2. What is the distance between A and B if they are thrown along the same straight line?

17. A boat traveling east covers a distance of 40.0 m in 20.0 s. It encounters a current moving at

a speed of 2.50 m/s traveling north. Find the resultant velocity of the boat. 18. A riverboat travels with a velocity of 4.60 m/s from one shore to another. The velocity of the

river is 2.30 m/s. If the width of the river is 72.0 m, how far does the boat travel downstream to reach the other shore?

19. A bus travels from west to east with a velocity of 11.5 m/s. A marble rolls on the surface of

the bus floor with a velocity of 0.200 m/s north. What is the velocity of the marble relative to the road?

20. Two boats, A and B, travel with a velocity of 4.90 m/s across a river of width 72.0 m. The

river flows with a velocity of 2.50 m/s. Boat A travels the shortest distance and boat B travels in the shortest time. If both start at the same time, how much time will they take to cross the river?

21. Tracy swims across a stream of width 40.0 m in 33.0 s when there is no current. She takes

59.0 s to cover the same distance when there is a current. Find the speed of the river current.

Page 77: Physics%–%1st%Quarter% - Lorain City School District · PDF file11 L12text’ ... Solving’ Many’ problems ... predict’the’effects’of’forces’onmore’complex’systems’of’objects’and’to’analyze’objects

Chapter 6 ANSWER KEY

MULTIPLE CHOICES

1. ANS: D 2. ANS: D

3. ANS: C

4. ANS: C

5. ANS: B

6. ANS: B

7. ANS: C

8. ANS: D

9. ANS: A 10. ANS: A

11. ANS: B

12. ANS: D SHORT ANSWER 13. ANS:

An object in uniform circular motion has a constant speed, but its velocity keeps changing. Since velocity is a vector quantity, a change in direction indicates a change in velocity. Since the velocity changes, the object is said to be accelerating.

PROBLEM 14. ANS: 38.8 m

15. ANS: 0.05 kg

16. ANS: 3.39 m.

17. ANS: The boat is traveling 3.20 m/s at 51.3 m/s north of east.

18. ANS: 36.0 m

Page 78: Physics%–%1st%Quarter% - Lorain City School District · PDF file11 L12text’ ... Solving’ Many’ problems ... predict’the’effects’of’forces’onmore’complex’systems’of’objects’and’to’analyze’objects

19. ANS: The marble is traveling 11.5 m/s at 1.00° north of east.

20. ANS: The time taken by boat A is 17.1 s. The time taken by boat B is 14.7 s.

21. ANS: 1.005 m/s

Chapter 6 ANSWER KEY ANALYSIS MULTIPLE CHOICES

2. ANS: D A projectile is any object that has been given an initial thrust and moves through air. Its path through space is called its trajectory. DIF: 1 REF: p.p. 147 OBJ: 6.1.1 Recognize that the vertical and horizontal motions of a projectile are independent. TOP: Recognize that the vertical and horizontal motions of a projectile are independent. KEY: Projectile, Trajectory MSC: 1 NOT: /a/ Equilibrant is a type of force. /b/ Torque is the product of the force and length of the lever arm. A projectile the horizontal distance travels /c/ Range. /d/ Correct!

3. ANS: D Draw a motion diagram for the trajectory showing the downward acceleration. The velocity will have two components, a horizontal and a vertical component. The combination of constant horizontal velocity and uniform vertical acceleration produces a distinct trajectory. DIF: 1 REF: p.p. 148 OBJ: 6.1.3 Explain how the shape of the trajectory of a projectile depends upon the frame of reference from which it is observed. TOP: Explain how the shape of the trajectory of a projectile depends upon the frame of reference from which it is observed. KEY: Projectile, Trajectory MSC: 1 NOT: /a/ a circular path would mean that the grenade falls on the soldier! /b/ the grenade cannot take an elliptical path, as gravity would act on it. /c/ the trajectory of the grenade cannot be a hyperbola as Earth would exert a gravitational pull on it. /d/ Correct!

4. ANS: C Solve the equation for the time the stone is in the air. Then solve the equation for the initial horizontal velocity by substituting the values of time and the range. DIF: 3 REF: p.p. 149

Page 79: Physics%–%1st%Quarter% - Lorain City School District · PDF file11 L12text’ ... Solving’ Many’ problems ... predict’the’effects’of’forces’onmore’complex’systems’of’objects’and’to’analyze’objects

OBJ: 6.1.2 Relate the height, time in the air, and initial vertical velocity of a projectile using its vertical motion, then determine the range using the horizontal motion.; TOP: Relate the height, time in the air, and initial vertical velocity of a projectile using its vertical motion, then determine the range using the horizontal motion. KEY: Projectiles launched horizontally MSC: 3 NOT: /a/ the time is 2.260 s. Divide the range by the time to get the initial horizontal velocity. /b/ Divide the range by the period, instead of multiplying the range by the period. /c/ Correct! /d/ you calculated the vertical velocity instead of the horizontal velocity.

5. ANS: C Use the equation for the y-position, and solve the equation for the time the stone is in the air. Multiply the time with the horizontal velocity to obtain the horizontal distance (range). DIF: 2 REF: p.p. 149 OBJ: 6.1.2 Relate the height, time in the air, and initial vertical velocity of a projectile using its vertical motion, then determine the range using the horizontal motion.; TOP: Relate the height, time in the air, and initial vertical velocity of a projectile using its vertical motion, then determine the range using the horizontal motion. KEY: Projectiles launched horizontally MSC: three NOT: /a/ Use the equation for the y-position to find the time period. /b/ Multiply the period with the horizontal velocity. /c/ Correct! /d/ the magnitude of the acceleration due to gravity is 9.80 m/s2.

6. ANS: B Use the equation for the y-position, and solve the equation for the time the stone is in the air. Multiply the time with the horizontal velocity to obtain the distance. DIF: 2 REF: p.p. 149 OBJ: 6.1.2 Relate the height, time in the air, and initial vertical velocity of a projectile using its vertical motion, then determine the range using the horizontal motion.; TOP: Relate the height, time in the air, and initial vertical velocity of a projectile using its vertical motion, then determine the range using the horizontal motion. KEY: Projectiles launched horizontally MSC: 3 NOT: /a/ you calculated the time the stone is in the air. Multiply this value with the initial horizontal velocity to find the range. /b/ Correct! /c/ Use the equation for the y-position to calculate how long the stone is in the air. /d/ to calculate the time the stone is in the air, use the formula t2 = –2 y/g.

7. ANS: B Write expressions for the vertical and horizontal velocity components. Solve the velocity equation for the time of maximum height. Substitute this time into the equation for vertical position to find the height.

Page 80: Physics%–%1st%Quarter% - Lorain City School District · PDF file11 L12text’ ... Solving’ Many’ problems ... predict’the’effects’of’forces’onmore’complex’systems’of’objects’and’to’analyze’objects

DIF: 3 REF: p.p. 150 OBJ: 6.1.2 Relate the height, time in the air, and initial vertical velocity of a projectile using its vertical motion, then determine the range using the horizontal motion.; TOP: Relate the height, time in the air, and initial vertical velocity of a projectile using its vertical motion, then determine the range using the horizontal motion. KEY: Maximum range MSC: 3 NOT: /a/you calculated the vertical component instead of the horizontal component. /b/ Correct! /c/ the actual velocity is always more than its horizontal component. /d/ Can the horizontal component of the velocity be more than the actual velocity?

8. ANS: C Write the equations for the initial velocity components, the velocity components at time t, and the position in both directions. DIF: 2 REF: p.p. 150 OBJ: 6.1.2 Relate the height, time in the air, and initial vertical velocity of a projectile using its vertical motion, then determine the range using the horizontal motion.; TOP: Relate the height, time in the air, and initial vertical velocity of a projectile using its vertical motion, then determine the range using the horizontal motion. KEY: Maximum height MSC: 2 NOT: /a/ did you multiply the velocity with the cosine of the angle instead of the sine? /b/ the time taken to reach maximum height can be calculated using the formula t = vyo/g, where vyo is the vertical component of the velocity. /c/ Correct! /d/ Substitute the vertical component of the velocity in the equation for the maximum height.

9. ANS: D Solve the vertical-position equation for the time at the end of the flight, when y = 0. Substitute that value of time into the equation for horizontal distance to get the range. DIF: 2 REF: p.p. 150 OBJ: 6.1.2 Relate the height, time in the air, and initial vertical velocity of a projectile using its vertical motion, then determine the range using the horizontal motion.; TOP: Relate the height, time in the air, and initial vertical velocity of a projectile using its vertical motion, then determine the range using the horizontal motion. KEY: Maximum range MSC: 2 NOT: /a/ Solve the vertical-position equation for the time at the end of the flight when y = 0. /b/ to find the range, multiply the horizontal component of velocity with the time obtained when y = 0. /c/ the vertical component of velocity is the product of the velocity and the sine of the angle the football makes with the horizontal. /d/ Correct!

10. ANS: A Uniform circular motion is the movement of an object or point mass at a constant speed around a circle with a fixed radius.

Page 81: Physics%–%1st%Quarter% - Lorain City School District · PDF file11 L12text’ ... Solving’ Many’ problems ... predict’the’effects’of’forces’onmore’complex’systems’of’objects’and’to’analyze’objects

DIF: 1 REF: p.p. 153 OBJ: 6.2.1 Explain why an object moving in a circle at constant speed is accelerated. TOP: Explain why an object moving in a circle at constant speed is accelerated. KEY: Uniform circular motion MSC: 1 NOT: /a/ Correct! /b/ a parabola is not a circle. /c/ an elliptical orbit does not have a fixed radius. /d/ a rotating object moves around its own vertical axis.

11. ANS: A

Substitute the values of velocity and radius in the equation for centripetal acceleration. DIF: 1 REF: p.p. 153 OBJ: 6.2.2 Describe how centripetal acceleration depends upon the object's speed and the radius of the circle. TOP: Describe how centripetal acceleration depends upon the object's speed and the radius of the circle. KEY: Centripetal acceleration MSC: 2 NOT: /a/ Correct! /b/ Divide the square of the velocity by the radius to calculate centripetal acceleration. /c/ Square the velocity in the formula for centripetal acceleration. /d/ Divide the square of the velocity by the radius.

12. ANS: B

Use Newton’s second law, .

DIF: 2 REF: p.p. 154 OBJ: 6.2.3 Identify the force that causes centripetal acceleration. TOP: Recognize the direction of the force that causes centripetal acceleration. KEY: Centripetal force MSC: 2 NOT: /a/ did you include the value of radius in the formula? /b/ Correct! /c/ Use Newton's second law. /d/ multiplies the mass with the square of the velocity and divides it by the radius.

13. ANS: D Substitute the value of velocity and radius in the equation for centripetal acceleration. DIF: 2 REF: p.p. 154 OBJ: 6.2.2 Describe how centripetal acceleration depends upon the object's speed and the radius of the circle. TOP: Describe how centripetal acceleration depends upon the object's speed and the radius of the circle. KEY: Centripetal acceleration MSC: 2 NOT: /a/ Divide the square of the velocity by the radius to get the centripetal acceleration. /b/ did you include the value of radius in the formula? /c/ Square the velocity in the formula. /d/ Correct!

Page 82: Physics%–%1st%Quarter% - Lorain City School District · PDF file11 L12text’ ... Solving’ Many’ problems ... predict’the’effects’of’forces’onmore’complex’systems’of’objects’and’to’analyze’objects

SHORT ANSWER 14. ANS: An object in uniform circular motion has a constant speed, but its velocity keeps changing. Since velocity is a vector quantity, a change in direction indicates a change in velocity. Since the velocity changes, the object is said to be accelerating. DIF: 3 REF: p.p. 153 OBJ: 6.2.1 Explain why an object moving in a circle at constant speed is accelerated. TOP: Explain why an object moving in a circle at constant speed is accelerated. KEY: Uniform circular motion MSC: 2 PROBLEM 15. ANS: 38.8 m DIF: 3 REF: p.p. 150 OBJ: 6.1.2 Relate the height, time in the air, and initial vertical velocity of a projectile using its vertical motion, then determine the range using the horizontal motion.; TOP: Relate the height, time in the air, and initial vertical velocity of a projectile using its vertical motion, then determine the range using the horizontal motion. KEY: Projectiles launched at an angle MSC: 3 NOT: Solve the y-equation for y = 0 to find the time the ball is in the air. Multiply the time with the horizontal component of the velocity to obtain the range.

16. ANS: 0.05 kg DIF: 3 REF: p.p. 154 OBJ: 6.2.3 Identify the force that causes centripetal acceleration. TOP: Identify the force that causes centripetal acceleration. KEY: Centripetal force MSC: 3 NOT: Find the velocity of the metallic object. The centripetal force for the metallic object is the product of the ratio and the centripetal force. Substitute this value in the relation F = mv^2/r to get the mass.

17. ANS: 3.39 m. DIF: 3 REF: p.p. 149 OBJ: 6.1.2 Relate the height, time in the air, and initial vertical velocity of a projectile using its vertical motion, then determine the range using the horizontal motion.; TOP: Relate the height, time in the air, and initial vertical velocity of a projectile using its vertical motion, then determine the range using the horizontal motion. KEY: Projectile, Trajectory MSC: 3 NOT: Use the equation for the y-position to solve the equation for the time the objects are in the air.

Page 83: Physics%–%1st%Quarter% - Lorain City School District · PDF file11 L12text’ ... Solving’ Many’ problems ... predict’the’effects’of’forces’onmore’complex’systems’of’objects’and’to’analyze’objects

18. ANS: The boat is traveling 3.20 m/s at 51.3 m/s north of east. DIF: 2 REF: p.p. 157 OBJ: 6.3.2 Solve relative-velocity problems. TOP: Solve relative-velocity problems. KEY: Relative velocity MSC: 3 NOT: Use the equation for the relative velocity of an object.

19. ANS: 36.0 m DIF: 3 REF: p.p. 157 OBJ: 6.3.2 Solve relative-velocity problems. TOP: Solve relative-velocity problems. KEY: Relative velocity MSC: 3 NOT: While calculating the time taken to reach the other shore, divide the width of the river by the boat velocity. While calculating the distance, multiply the time by the river velocity.

20. ANS: The marble is traveling 11.5 m/s at 1.00° north of east.

DIF: 2 REF: Page 157 OBJ: 6.3.2 Solve relative-velocity problems. TOP: Solve relative-velocity problems. KEY: Relative velocity MSC: 3 NOT: Use the equation for the relative velocity of an object.

21. ANS: The time taken by boat A is 17.1 s. The time taken by boat B is 14.7 s.

DIF: 3 REF: Page 157 OBJ: 6.3.2 Solve relative-velocity problems. TOP: Solve relative-velocity problems. KEY: Relative velocity MSC: 3 NOT: Use the equation for relative velocity of an object.

22. ANS: 1.005 m/s DIF: 3 REF: Page 157 OBJ: 6.3.2 Solve relative-velocity problems. TOP: Solve relative-velocity problems. KEY: Relative velocity MSC: 3 NOT: Use the equation to calculate the relative velocity of an object.

*END OF EXAMS 4 ->6*

Page 84: Physics%–%1st%Quarter% - Lorain City School District · PDF file11 L12text’ ... Solving’ Many’ problems ... predict’the’effects’of’forces’onmore’complex’systems’of’objects’and’to’analyze’objects

Name ___________________________________________________ Period _________________

Chapter 7 Test

PROBLEM 1. The distance between Pluto and the Sun is 39.1 times more than the distance between the Sun

and Earth. Calculate the time taken by Pluto to orbit the Sun in Earth days. 2. The Moon has an orbital period of 27.3 days around Earth and a mean distance of 3.85 x 105

km from Earth’s center. Use Kepler’s laws to find the orbital period of an artificial satellite orbiting Earth at a distance of km from the center of Earth.

3. Venus orbits the Sun with an orbital radius of m. Given that the mass of the Sun is

2.0 x 1030 kg, calculate the period of Venus’s orbit. 4. Jupiter orbits the Sun with an orbital radius of m. Given that the mass of the Sun is 2.00 x 1030 kg, calculate the orbital velocity of Jupiter. 5. Calculate the force of gravitational attraction between two spheres of mass 10.1 kg and 45.4

kg that are 38.5 m apart. 6. Venus has radius m and mass kg. Calculate the value of acceleration

due to gravity on Venus’s surface. 7. If Earth shrinks in size such that its shape and mass remain the same, but the radius

decreases to 0.21 times its original value, find the acceleration due to gravity on its surface. 8. A satellite orbits Neptune 4000 km above its surface. Given that the mass of Neptune is

kg and the radius of Neptune is m, calculate the orbital speed of the satellite.

9. A satellite orbits Jupiter km above its surface. Given that the mass of Jupiter is

kg and the radius of Jupiter is m, calculate the period of orbit of the satellite.

10. At what height above Earth’s surface does the gravitational intensity becomes 0.67 times its

value on the surface of Earth? Given the radius of the Earth is 6.40 x 106 m.

3.03× 104

1.08× 1011

7.78× 1011

6.05× 106 4.87× 1024

1.02× 1026 2.48× 107

6.40× 103

1.90× 1027 7.15× 107

Page 85: Physics%–%1st%Quarter% - Lorain City School District · PDF file11 L12text’ ... Solving’ Many’ problems ... predict’the’effects’of’forces’onmore’complex’systems’of’objects’and’to’analyze’objects

Chapter7 ANSWER KEY PROBLEM

1. ANS: 89300 Earth days

2. ANS: 0.603 days

3. ANS: s

4. ANS: 13.1 km/s

5. ANS: N 6. ANS: 8.9 m/s2

7. ANS: 220 m/s2

8. ANS: 15 km/s

9. ANS: s

10. ANS: 1400 km

1.9× 107

2.06× 10−11

1.21× 104

Page 86: Physics%–%1st%Quarter% - Lorain City School District · PDF file11 L12text’ ... Solving’ Many’ problems ... predict’the’effects’of’forces’onmore’complex’systems’of’objects’and’to’analyze’objects

Chapter7 ANSWER KEY ANALYSIS PROBLEM

1. ANS: 89300 Earth days DIF: 3 REF: p.p. 173 OBJ: 7.1.1 Relate Kepler's laws to the law of universal gravitation. TOP: Relate Kepler's laws to the law of universal gravitation. KEY: Kepler's laws MSC: 3 NOT: The square of the period of a planet's orbit around the Sun is proportional to the cube of its mean distance from the Sun.

2. ANS: 0.603 days DIF: 3 REF: p.p. 173 OBJ: 7.1.1 Relate Kepler's laws to the law of universal gravitation. TOP: Relate Kepler's laws to the law of universal gravitation. KEY: Kepler's laws MSC: 3 NOT: The square of the period of a planet's orbit around the Sun is proportional to the cube of its mean distance from the Sun.

3. ANS: s DIF: 3 REF: p.p. 176 OBJ: 7.1.2 Calculate orbital speeds and periods. TOP: Calculate orbital speeds and periods. KEY: Orbital period MSC: 3 NOT: The square of the period of a planet's orbit around the Sun is proportional to the cube of its mean distance from the Sun.

4. ANS: 13.1 km/s DIF: 3 REF: p.p. 176 OBJ: 7.1.2 Calculate orbital speeds and periods. TOP: Calculate orbital speeds and periods. KEY: Orbital speed MSC: 3 NOT: The orbital velocity of a planet is inversely proportional to the square root of its radius of orbit around the Sun.

5. ANS: N DIF: 3 REF: p.p. 177, p.p. 178 OBJ: 7.1.3 Describe the importance of Cavendish's experiment.

1.9× 107

2.06× 10−11

Page 87: Physics%–%1st%Quarter% - Lorain City School District · PDF file11 L12text’ ... Solving’ Many’ problems ... predict’the’effects’of’forces’onmore’complex’systems’of’objects’and’to’analyze’objects

TOP: Describe the importance of Cavendish's experiment. KEY: Gravitational force MSC: 3 NOT: Use the mathematical form of Newton's law of gravitation.

6. ANS: 8.9 m/s2

DIF: 3 REF: p.p. 177, p.p. 178 OBJ: 7.1.3 Describe the importance of Cavendish's experiment. TOP: Describe the importance of Cavendish's experiment. KEY: Acceleration due to gravity MSC: 3 NOT: Acceleration due to gravity is directly proportional to the mass of the planet and inversely proportional to the square of the planet's radius.

7. ANS: 220 m/s2 DIF: 3 REF: p.p. 177, p.p. 178 OBJ: 7.1.3 Describe the importance of Cavendish's experiment. TOP: Describe the importance of Cavendish's experiment. KEY: Acceleration due to gravity MSC: 3 NOT: Acceleration due to gravity is directly proportional to the mass of the planet and inversely proportional to the square of the planet's radius.

8. ANS: 15 km/s DIF: 3 REF: p.p. 180 OBJ: 7.2.1 Solve orbital motion problems. TOP: Solve orbital motion problems. KEY: Orbital velocity MSC: 3 NOT: The orbital velocity of a satellite is directly proportional to the square root of the planet's mass and inversely proportional to the square root of the satellite's distance from the center of the planet.

9. ANS: s DIF: 3 REF: p.p. 180 OBJ: 7.2.1 Solve orbital motion problems. TOP: Solve orbital motion problems. KEY: Orbital period MSC: 3 NOT: The orbital period of a satellite is directly proportional to the square root of the cube of the satellite's distance from the center of the planet and inversely proportional to the square root of the planet's mass.

10. ANS: 1400 km DIF: 3 REF: p.p. 182, p.p. 183 OBJ: 7.2.3 Describe gravitational fields. TOP: Describe gravitational fields. KEY: Gravitational field

1.21× 104

Page 88: Physics%–%1st%Quarter% - Lorain City School District · PDF file11 L12text’ ... Solving’ Many’ problems ... predict’the’effects’of’forces’onmore’complex’systems’of’objects’and’to’analyze’objects

MSC: 3 NOT: The gravitational intensity of a planet is inversely proportional to the square of the distance from Earth's center.

Name ___________________________________________________ Period _________________

Chapter 8 Test

PROBLEM 1. A disc of radius 5.70 cm rotates about its axis and a point 1.90 cm from the center of the disc

moves 34.5 cm in 12.2 s. Calculate the angular speed of the disc. 2. A car wheel turns through 277° in 10.7 s. Calculate the angular speed of the wheel.

3. A torque acts on a wheel rotating at 19.8 rad/s and increases its angular speed to 23.5 rad/s in 11.2 s. Find the angle through which the wheel turns during this time. 4. A disc rotates at 5.60 rad/s. An angular acceleration acts on the disc to change its angular

speed to 16.8 rad/s. If the disc turns through 12.2 rad during this time, calculate the angular acceleration of the disc.

5. A car travels at 1.70 m/s. The driver accelerates and increases the speed to 10.7 m/s in 2.20 s.

If the radius of its wheel is 0.590 m, calculate the angle turned by the wheel during this time. 6. A beam that weighs 10.0 N/m is 2.5 m long. It is supported at a point 0.78 m from one end.

Find the weight of the object that must be placed on the other end of the beam to balance it. 7. A worker sits at one end of a 183-N uniform rod that is 2.80 m long. A weight of 107 N is

placed at the other end of the rod. The rod is balanced when the pivot is 0.670 m from the worker. Calculate the weight of the worker.

8. A rod of length 2.4 m is pivoted at its center and masses of 2.3 kg and 3.1 kg are hung from

its two ends. Find the initial angular acceleration of the rod when the system is released from a starting position of the rod that is horizontal.

9. Calculate the net torque on the rectangular plate about the point O. Force F1 is 12.0 N, F2 is

21.0 N, F3 is 12.0 N, F4 is 16.0 N, and F5 is 19.0 N.

Page 89: Physics%–%1st%Quarter% - Lorain City School District · PDF file11 L12text’ ... Solving’ Many’ problems ... predict’the’effects’of’forces’onmore’complex’systems’of’objects’and’to’analyze’objects

10. A 1.29-m long ladder weighing 200 N rests against a vertical wall so that the top of the

ladder is at a height of 0.590 m. A 603-N man stands on the ladder at a distance of 0.390 m along the ladder. Calculate the force exerted by the wall on the ladder. Assume that the wall is perfectly smooth.

Page 90: Physics%–%1st%Quarter% - Lorain City School District · PDF file11 L12text’ ... Solving’ Many’ problems ... predict’the’effects’of’forces’onmore’complex’systems’of’objects’and’to’analyze’objects

Chapter 8 ANSWER KEY

PROBLEM

1. ANS: 1.49 rad/s

2. ANS: 0.452 rad/s

3. ANS: 243 rad

4. ANS: 10.3 rad/s2

5. ANS: 23.1 rad 6. ANS: 15 N

7. ANS: 549 N

8. ANS: 0.50 rad/s2

9. ANS: 73 N·m

10. ANS: 617 N

Page 91: Physics%–%1st%Quarter% - Lorain City School District · PDF file11 L12text’ ... Solving’ Many’ problems ... predict’the’effects’of’forces’onmore’complex’systems’of’objects’and’to’analyze’objects

Chapter 8

ANSWER KEY ANALYSIS PROBLEM

1. ANS: 1.49 rad/s DIF: 3 REF: p.p., p.p. 199 OBJ: 8.1.2 Calculate angular velocity. TOP: Calculate angular velocity. KEY: Angular velocity MSC: 3 NOT: The angular speed of a rotating body is equal to the linear speed of a point divided by the distance of the point from the axis of rotation.

2. ANS: 0.452 rad/s DIF: 3 REF: p.p. 198, p.p. 199 OBJ: 8.1.2 Calculate angular velocity. TOP: Calculate angular velocity. KEY: Angular velocity MSC: 3 NOT: The angular speed of a rotating body is equal to the linear speed of a point divided by the distance of the point from the axis of rotation.

3. ANS: 243 rad DIF: 3 REF: p.p. 198, p.p. 199 OBJ: 8.1.4 Solve problems involving rotational motion. TOP: Solve problems involving rotational motion. KEY: Rotational motion MSC: 3 NOT: The angle turned is the product of the average angular speed and the time.

4. ANS: 10.3 rad/s2 DIF: 3 REF: p.p. 198, p.p. 199 OBJ: 8.1.4 Solve problems involving rotational motion. TOP: Solve problems involving rotational motion. KEY: Rotational motion MSC: 3 NOT: The angular acceleration is equal to the difference between the squares of the final and initial angular speeds divided by twice the angle turned.

5. ANS: 23.1 rad DIF: 3 REF: p.p. 198, p.p. 199 OBJ: 8.1.4 Solve problems involving rotational motion. TOP: Solve problems involving rotational motion. KEY: Rotational motion

Page 92: Physics%–%1st%Quarter% - Lorain City School District · PDF file11 L12text’ ... Solving’ Many’ problems ... predict’the’effects’of’forces’onmore’complex’systems’of’objects’and’to’analyze’objects

MSC: 3 NOT: The angle turned is equal to the distance traveled by the car divided by the radius of the wheel. 6. ANS: 15 N DIF: 3 REF: p.p. 202, p.p. 203 OBJ: 8.2.2 Calculate net torque. TOP: Calculate net torque. KEY: Net torque MSC: 3 NOT: The torque due to the weight of the beam is equal to the torque due to the weight required to balance the beam.

7. ANS: 549 N DIF: 3 REF: p.p. 202, p.p. 203 OBJ: 8.2.2 Calculate net torque. TOP: Calculate net torque. KEY: Net torque MSC: 3 NOT: The torque due to the weight of the beam and the required weight is equal to the torque due to the man's weight.

8. ANS: 0.50 rad/s2 DIF: 3 REF: p.p. 202, p.p. 203 OBJ: 8.2.2 Calculate net torque. TOP: Calculate net torque. KEY: Net torque MSC: 3 NOT: The net torque is equal to the difference between the torque due to the weight of each mass. The angular acceleration is equal to the torque divided by the total moment of inertia.

9. ANS: 73 N·m DIF: 3 REF: p.p. 202, p.p. 203 OBJ: 8.2.2 Calculate net torque. TOP: Calculate net torque. KEY: Net torque MSC: 3 NOT: The net torque is the algebraic sum of the product of each force and its perpendicular distance from O.

10. ANS: 617 N DIF: 3 REF: p.p. 213 OBJ: 8.3.3 Define the conditions for equilibrium. TOP: Define the conditions for equilibrium. KEY: Equilibrium MSC: 3 NOT: Because the ladder is in equilibrium, the net force must be equal to zero and the net torque must be equal to zero.

*End of exams 7+8*

Page 93: Physics%–%1st%Quarter% - Lorain City School District · PDF file11 L12text’ ... Solving’ Many’ problems ... predict’the’effects’of’forces’onmore’complex’systems’of’objects’and’to’analyze’objects

Name ___________________________________________________ Period _________________

Chapter 9 Test PROBLEM 1. A 6110-kg bus traveling at 20.0 m/s can be stopped in 24.0 s by gently applying the brakes.

If the driver slams on the brakes, the bus stops in 3.90 s. What is the average force exerted on the bus in both these stops?

2. A 0.140-kg baseball is pitched horizontally at 36.7 m/s. When a player hits the ball, it moves

at the same speed, but in the opposite direction. If the bat and the ball are in contact for 0.450 ms, calculate the average force the bat exerts on the ball.

3. Candona strikes a 0.055-kg golf ball with a force of 260 N. If the ball moves with a velocity

of 65 m/s, calculate the time the ball is in contact with the club. 4. A force of 200 N acts on a 7.20-kg bowling ball for 0.350 s. Calculate its change in velocity. 5. The moment of inertia of an asteroid rotating about its own axis is kg m2. Its angular

velocity is 40.0 rad/s. If a force acts on the asteroid for 0.100 s, increasing the angular velocity to 48.0 rad/s, find its magnitude.

6. A toy car X of mass 0.200 kg moves along a frictionless surface with a velocity of 0.180 m/s.

It collides with another toy car Y, with a mass of 0.250 kg and a speed of 0.130 m/s in the same direction. After the collision, toy car X continues to move in the same direction with a velocity of 0.177 m/s. Calculate the speed of toy car Y after the collision.

7. A marksman at rest fires a 4.00-kg gun that expels a bullet of mass 0.0140 kg with a velocity

of 181 m/s. The marksman’s mass is 81.0 kg. What is the marksman’s velocity after firing the gun?

8. A rocket expels gases at a rate of kg/s with a speed of m/s. What is the

force exerted on the rocket? 9. Ball A, with a mass of 1.75 kg, moves with a velocity 3.50 m/s. It collides with a stationary

ball B, with a mass of 2.50 kg. After the collision, ball A moves in a direction 55.0 north to the left of its original direction, while ball B moves in a direction 35.0 south to the right of ball A’s original direction. Calculate the velocity of each ball after the collision.

5.00× 104

1.30× 103 3.00× 104

Page 94: Physics%–%1st%Quarter% - Lorain City School District · PDF file11 L12text’ ... Solving’ Many’ problems ... predict’the’effects’of’forces’onmore’complex’systems’of’objects’and’to’analyze’objects

Chapter 9 ANSWER KEY

PROBLEM 1. ANS:

N

N

2. ANS: N

3. ANS: 0.014 s

4. ANS: 9.72 m/s

5. ANS: N

6. ANS: 0.132 m/s

7. ANS: 0.0298 m/s

8. ANS: N

9. ANS: The velocity of ball A is 2.01 m/s. The velocity of ball B is 2.01 m/s.

5.09× 103

3.13× 104

2.28× 104

4.00× 106

3.90× 107

Page 95: Physics%–%1st%Quarter% - Lorain City School District · PDF file11 L12text’ ... Solving’ Many’ problems ... predict’the’effects’of’forces’onmore’complex’systems’of’objects’and’to’analyze’objects

Chapter 9

ANSWER KEY ANALYSIS PROBLEM 2. ANS:

N

N

DIF: 3 REF: p.p. 230 OBJ: 9.1.2 Determine the impulse given to an object. TOP: Determine the impulse given to an object. KEY: Impulse MSC: 3 NOT: Apply the impulse-momentum theorem to obtain the force needed to stop the vehicle.

2. ANS: N DIF: 3 REF: p.p. 230 OBJ: 9.1.2 Determine the impulse given to an object. TOP: Determine the impulse given to an object. KEY: Impulse MSC: 3 NOT: Apply the impulse-momentum theorem to obtain the force the bat exerts on the ball.

3. ANS: 0.014 s DIF: 3 REF: p.p. 230 OBJ: 9.1.2 Determine the impulse given to an object. TOP: Determine the impulse given to an object. KEY: Impulse MSC: 3 NOT: Apply the impulse-momentum theorem to obtain the time the ball is in contact with the club.

4. ANS: 9.72 m/s DIF: 3 REF: p.p. 230 OBJ: 9.1.2 Determine the impulse given to an object. TOP: Determine the impulse given to an object. KEY: Impulse MSC: 3 NOT: Apply the impulse-momentum theorem to obtain the change in velocity.

5. ANS: N DIF: 3 REF: p.p. 234 OBJ: 9.1.3 Define the angular momentum of an object. TOP: Define the angular momentum of an object. KEY: Angular momentum MSC: 3 NOT: Apply the angular impulse-angular momentum theorem to find the angular impulse.

6. ANS: 0.132 m/s

5.09× 103

3.13× 104

2.28× 104

4.00× 106

Page 96: Physics%–%1st%Quarter% - Lorain City School District · PDF file11 L12text’ ... Solving’ Many’ problems ... predict’the’effects’of’forces’onmore’complex’systems’of’objects’and’to’analyze’objects

DIF: 3 REF: p.p. 237 OBJ: 9.2.1 Relate Newton's third law to conservation of momentum. TOP: Relate Newton's third law to conservation of momentum. KEY: Conservation of momentum, Newton's third law MSC: 3 NOT: Apply Newton's third law and the law of conservation of momentum.

7. ANS: 0.0298 m/s DIF: 3 REF: p.p. 240 OBJ: 9.2.3 Solve conservation of momentum problems. TOP: Solve conservation of momentum problems. KEY: Conservation of momentum MSC: 3 NOT: Apply the law of conservation of momentum.

8. ANS: N DIF: 3 REF: p.p. 237 OBJ: 9.2.1 Relate Newton's third law to conservation of momentum. TOP: Relate Newton's third law to conservation of momentum. KEY: Conservation of momentum MSC: 3 NOT: Apply the law of conservation of momentum.

9. ANS: The velocity of ball A is 2.01 m/s. The velocity of ball B is 2.01 m/s. DIF: 3 REF: p.p. 242 OBJ: 9.2.3 Solve conservation of momentum problems. TOP: Solve conservation of momentum problems. KEY: Conservation of momentum MSC: 3 NOT: Apply the law of conservation of momentum in two dimensions.

3.90× 107

Page 97: Physics%–%1st%Quarter% - Lorain City School District · PDF file11 L12text’ ... Solving’ Many’ problems ... predict’the’effects’of’forces’onmore’complex’systems’of’objects’and’to’analyze’objects

Name ___________________________________________________ Period _________________

Chapter 10 Test PROBLEM 1. Carol and Bruno drag a box of mass 58.0 kg along a frictionless floor. Carol pushes the box

with a force of 11.4 N at an angle of 40.0° downward from the horizontal. Bruno pulls the box from the other side with a force of 11.0 N at an angle of 40.0° above the horizontal. What is the net work done on the box if the displacement of the box is 14.5 m?

2. A cable pulls a stationary crate of mass 19.0 kg over a frictionless ramp at an angle 20.1°

above the ground. If the total distance traveled is 5.40 m, find the work done by the cable on the crate.

3. Raul pushes a stalled car with a force of 204 N. If the required force decreases at a constant rate from 204 N to 44.0 N for a distance of 16.3 m in 16.0 s, calculate the average power required to move the car.

4. A 1600-kg vehicle moves with a velocity of 19.5 m/s. Calculate the power required to reduce

the velocity to 3.20 m/s in 11.0 s. 5. Ayesha exerts a force of 186 N on a lever to raise a 2.70 103-N object to a height of 29.0

cm. If the efficiency of the lever is 83.3 percent, how far does Ayesha move her end of the lever?

6. A bicycle has a wheel of radius 43.4 cm and a gear of radius 5.20 cm. When the chain is

pulled with a force of 152.4 N, the wheel rim moves 15.7 cm. If the efficiency of the bicycle is 92.6 percent, how far was the chain pulled to move the rim 15.7 cm? Also, find the resistance force.

7. Aryton uses a pulley system to raise a 23.1-kg block to a height of 18.0 m. If a force of 124 N

is exerted and the rope is pulled 34.2 m, find the efficiency of the system.

8. A worker exerts a force of 119 N on a lever to raise a block. The efficiency of the lever is 91.6 percent. If the lever is moved 0.770 m, the block rises to a height of 0.180 m. What is the mass of the block?

Page 98: Physics%–%1st%Quarter% - Lorain City School District · PDF file11 L12text’ ... Solving’ Many’ problems ... predict’the’effects’of’forces’onmore’complex’systems’of’objects’and’to’analyze’objects

Chapter10 ANSWER KEY PROBLEM

1. ANS: J

2. ANS: J

3. ANS: 126 W 4. ANS: W

5. ANS: 5.05 m

6. ANS: The distance is 1.88 cm. The resistance force is 16.9 N.

7. ANS: 95.8 percent

8. ANS: 47.6 kg

2.49× 102

3.46× 102

2.69× 104

Page 99: Physics%–%1st%Quarter% - Lorain City School District · PDF file11 L12text’ ... Solving’ Many’ problems ... predict’the’effects’of’forces’onmore’complex’systems’of’objects’and’to’analyze’objects

Chapter 10 ANSWER KEY ANALYSIS PROBLEM

1. ANS: J DIF: 3 REF: p.p. 260 OBJ: 10.1.2 Calculate work. TOP: Calculate work. KEY: Work MSC: 3 NOT: Work is equal to the product of force and displacement times the cosine of the angle between the force and the direction of the displacement.

2. ANS: J DIF: 3 REF: p.p. 260 OBJ: 10.1.2 Calculate work. TOP: Calculate work. KEY: Work MSC: 3 NOT: Use the equation for work when a constant force is exerted in the same direction as the object's displacement. Since the object is moving against gravity, find the height of the object and substitute in the equation.

3. ANS: 126 W DIF: 3 REF: p.p. 263 OBJ: 10.1.3 Calculate the power used. TOP: Calculate the power used. KEY: Power MSC: 3 NOT: Power is equal to the work done divided by the time taken to do the work. 4. ANS: W DIF: 3 REF: p.p. 263 OBJ: 10.1.3 Calculate the power used. TOP: Calculate the power used. KEY: Power MSC: 3 NOT: Work is equal to the change in kinetic energy. Power is equal to the work done divided by the time taken to do the work.

5. ANS: 5.05 m DIF: 3 REF: p.p. 271 OBJ: 10.2.4 Calculate efficiencies for simple and compound machines. TOP: Calculate efficiencies for simple and compound machines. KEY: Efficiency MSC: 3 NOT: Use the formula for efficiency of a machine.

6. ANS: The distance is 1.88 cm. The resistance force is 16.9 N.

2.49× 102

3.46× 102

2.69× 104

Page 100: Physics%–%1st%Quarter% - Lorain City School District · PDF file11 L12text’ ... Solving’ Many’ problems ... predict’the’effects’of’forces’onmore’complex’systems’of’objects’and’to’analyze’objects

DIF: 3 REF: p.p. 271 OBJ: 10.2.4 Calculate efficiencies for simple and compound machines. TOP: Calculate efficiencies for simple and compound machines. KEY: Mechanical advantage, Ideal mechanical advantage, Efficiency MSC: 3 NOT: For a wheel-and-axle machine, IMA is equal to the ratio of radii.

7. ANS: 95.8 percent DIF: 3 REF: p.p. 271 OBJ: 10.2.4 Calculate efficiencies for simple and compound machines. TOP: Calculate efficiencies for simple and compound machines. KEY: Efficiency MSC: 3 NOT: The efficiency of a machine is equal to the output work, divided by the input work, multiplied by 100.

8. ANS: 47.6 kg DIF: 3 REF: p.p. 271 OBJ: 10.2.4 Calculate efficiencies for simple and compound machines. TOP: Calculate efficiencies for simple and compound machines. KEY: Efficiency MSC: 3 NOT: Use the formula for efficiency of a machine.

Page 101: Physics%–%1st%Quarter% - Lorain City School District · PDF file11 L12text’ ... Solving’ Many’ problems ... predict’the’effects’of’forces’onmore’complex’systems’of’objects’and’to’analyze’objects

Name ___________________________________________________ Period _________________

Chapter 11 PROBLEM 1. Andrew throws a 0.11-kg ball toward Donald, who is standing on a ledge. The ball leaves

Andrew’s hands at a height of 0.24 m and Donald catches it at a height of 0.82 m. Calculate the gravitational potential energy of the ball relative to the ground before being thrown.

2. A warehouse worker pushed a cart weighing 4.50 kg to the top of an inclined plane. Initially,

the cart was 0.670 m above the floor. If the top of the inclined plane is 2.70 m above the floor,

calculate the work done by gravity as the worker pushed the cart to the top of the plane. 3. A student lifts a 1.2-kg bag from her desk, which is 0.59-m high, to a locker that is 2.9-m

high. What is the gravitational potential energy of the bag relative to the desk? 4. A hemispherical bowl of radius 0.14 m is kept inverted on a tabletop. A 4.3-g matchbox slides

off from rest from the top of the hemisphere. What is the speed of the matchbox when it reaches a point on the bowl at an angle of 16.0° to the horizontal?

5. A skier is pushed from the top of a hill so that he starts moving down the hillside sloped at

27.6° to the horizontal with an initial speed of 0.434 m/s. After traveling 80.4 m, he reaches the bottom of the valley. Due to inertia, he then continues 70.4 m up another hillside sloped at 20.7° to the horizontal. What is the skier’s speed when he reaches the top of the hill? Assume that you can neglect friction.

6. A roller coaster car starts with an initial speed of 0.375 m/s from the top of a section of a

track that is sloping down at 26.2° to the horizontal. After traveling 80.3 m, it reaches the end of that section of the track and continues up the next section of the track that slopes up at 22.4° to the horizontal. If this section of the track has a length of 73.2 m, will the car reach the top of the section? Assume that you can neglect friction and that the car has no source of power.

7. A -kg bullet is fired with a velocity of 154 m/s toward a 5.44-kg stationary solid

block resting on a surface that has a coefficient of friction 0.215. The bullet emerges with a reduced velocity of 20.2 m/s after passing through the block. What distance will the block slide before coming to rest? Assume that the block does not lose any mass.

2.90× 10−3

Page 102: Physics%–%1st%Quarter% - Lorain City School District · PDF file11 L12text’ ... Solving’ Many’ problems ... predict’the’effects’of’forces’onmore’complex’systems’of’objects’and’to’analyze’objects

8. A -kg bullet is fired with a velocity of 108.6 m/s toward a 5.99-kg block moving at a speed of 0.280 m/s in the same direction. The bullet emerges with a speed of 20.7 m/s and with a small piece of the block with a mass of 0.0016 kg sticking to it. What is the kinetic energy lost in the collision?

9. A 0.390-kg piece of wood is at rest on a frictionless table. A 8.30-g bullet, moving with a

speed of 495 m/s, strikes the piece of wood, and is embedded in it. After the collision, the piece of wood and the bullet move slowly down the table. What percentage of the system’s original kinetic energy was lost?

2.10× 10−3

Page 103: Physics%–%1st%Quarter% - Lorain City School District · PDF file11 L12text’ ... Solving’ Many’ problems ... predict’the’effects’of’forces’onmore’complex’systems’of’objects’and’to’analyze’objects

Chapter 11 ANSWER KEY

PROBLEM

1. ANS:

2. ANS: –89.5 J

3. ANS: 27 J

4. ANS: Speed of matchbox = 1.4 m/s 5. ANS: Speed of skier = 15.6 m/s

6. ANS: Yes. The car will reach the top of the next hill.

7. ANS: m

8. ANS: 12 J

9. ANS: 97.9 %

1.21× 10−3

Page 104: Physics%–%1st%Quarter% - Lorain City School District · PDF file11 L12text’ ... Solving’ Many’ problems ... predict’the’effects’of’forces’onmore’complex’systems’of’objects’and’to’analyze’objects

Chapter 11

ANSWER KEY ANALYSIS PROBLEM

1. ANS: DIF: 2 REF: p.p. 285 OBJ: 11.1.3 Determine the gravitational potential energy of a system. TOP: Determine the gravitational potential energy of a system. KEY: Gravitational potential energy MSC: 3 NOT: The gravitational potential energy of an object is equal to the product of its mass, the acceleration due to gravity, and its height from the reference level.

2. ANS: –89.5 J DIF: 2 REF: p.p. 285 OBJ: 11.1.3 Determine the gravitational potential energy of a system. TOP: Determine the gravitational potential energy of a system. KEY: Gravitational potential energy MSC: 3 NOT: The work done by gravity is the weight of the cart times the vertical distance through which the cart is pushed.

3. ANS: 27 J DIF: 2 REF: p.p. 285 OBJ: 11.1.3 Determine the gravitational potential energy of a system. TOP: Determine the gravitational potential energy of a system. KEY: Gravitational potential energy MSC: 3 NOT: The gravitational potential energy of an object is equal to the product of its mass, the acceleration due to gravity, and its height from the reference level.

4. ANS: Speed of matchbox = 1.4 m/s DIF: 3 REF: p.p. 293 OBJ: 11.2.1 Solve problems using the law of conservation of energy. TOP: Solve problems using the law of conservation of energy. KEY: Law of conservation of energy MSC: 3 NOT: The kinetic energy gained by an object is equal to the gravitational potential energy lost by the object.

Page 105: Physics%–%1st%Quarter% - Lorain City School District · PDF file11 L12text’ ... Solving’ Many’ problems ... predict’the’effects’of’forces’onmore’complex’systems’of’objects’and’to’analyze’objects

5. ANS: Speed of skier = 15.6 m/s

DIF: 3 REF: p.p. 293 OBJ: 11.2.1 Solve problems using the law of conservation of energy. TOP: Solve problems using the law of conservation of energy. KEY: Law of conservation of energy MSC: 3 NOT: The kinetic energy gained by an object is equal to the gravitational potential energy lost by the object.

6. ANS: Yes. The car will reach the top of the next hill. DIF: 3 REF: p.p. 293 OBJ: 11.2.1 Solve problems using the law of conservation of energy. TOP: Solve problems using the law of conservation of energy. KEY: Law of conservation of energy MSC: 3 NOT: The gravitational potential energy gained by an object is equal to the kinetic energy lost by the object.

7. ANS: m DIF: 3 REF: p.p. 293 OBJ: 11.2.2 Analyze collisions to find the change in kinetic energy. TOP: Analyze collisions to find the change in kinetic energy. KEY: Collisions, Change in kinetic energy MSC: 3 NOT: The principle of conservation of momentum can be used to find the final velocity of the block. The kinetic energy of the block is used in doing work against friction.

8. ANS: 12 J DIF: 3 REF: p.p. 293 OBJ: 11.2.2 Analyze collisions to find the change in kinetic energy. TOP: Analyze collisions to find the change in kinetic energy. KEY: Collisions, Change in kinetic energy MSC: 3 NOT: The principle of conservation of momentum can be used to find the final velocity of the block without the missing part. The loss in kinetic energy is the difference between the initial total kinetic energy and the final total kinetic energy.

9. ANS: 97.9 % DIF: 3 REF: p.p. 293 OBJ: 11.2.2 Analyze collisions to find the change in kinetic energy. TOP: Analyze collisions to find the change in kinetic energy. KEY: Collisions MSC: 3 NOT: Use the conservation of momentum equation to find the final velocity. Determine the

1.21× 10−3

Page 106: Physics%–%1st%Quarter% - Lorain City School District · PDF file11 L12text’ ... Solving’ Many’ problems ... predict’the’effects’of’forces’onmore’complex’systems’of’objects’and’to’analyze’objects

change in kinetic energy of the system. Use the change in the kinetic energy to find the fraction of the original kinetic energy lost.

*End of Exams 9 -> 11*